Ihr seht also, dass von den drei anderen Kräften die elektromagnetische Wechselwirkung der Gravitation am ähnlichsten ist. Wir können also versuchen, eine Quantengravitation zu bauen, die der Quantentheorie des Elektromagnetismus (kurz QED für Quantenelektrodynamik genannt) ähnlich ist.

Die Masse des Gravitons

Wenn die QG ähnlich funktionieren soll, wie die QED, dann muss es ein Austauschteilchen geben. Dieses Teilchen ist das (hypothetische) Graviton. Falls es euch verwirrt, wie die Schwerkraft, die doch nach Einstein etwas mit der Raumzeitkrümmung zu tun haben soll, durch Teilchen beschrieben werden kann – das habe ich gerade ausführlich erklärt (wie gesagt, dieser Text und der über Gravitonen ergänzen einander).

Die Reichweite der Schwerkraft ist offensichtlich ziemlich groß – immerhin hält sie ganze Galaxienhaufen zusammen, die Millionen Lichtjahre groß sind. Daraus können wir – ähnlich wie beim Photon – schließen, dass die Masse des Gravitons extrem klein sein muss (siehe hier bei Andreas Müller für Details) – normalerweise geht man von einer Masse gleich null aus.

Der Spin des Gravitons

Elementarteilchen haben eine seltsame Eigenschaft namens Spin. In einem sehr einfachen Modell, in dem man die Teilchen wie kleine Kugeln betrachtet, kann man sich vorstellen, dass die Teilchen sich um ihre Achse drehen – das ist kein sehr gutes Bild, veranschaulicht aber zumindest, dass der Spin eine Größe ist, die etwas mit Drehungen zu tun hat. Schaut man genauer hin, dann ist der Spin ein Ausdruck dafür, wie sich Teilchen verhalten, wenn man sie dreht. Eine Kugel beispielsweise kann ich drehen wie ich will, sie sieht immer gleich aus. Einen Würfel kann ich entlang einer Achse senkrecht zu einer Fläche drehen – es reicht eine Drehung um 90°, damit der Würfel wieder so aussieht wie vorher. Der Spin sagt uns, wie wir ein Objekt drehen müssen, bis es wieder gleich aussieht. (Auch das habe ich mal ausführlich erklärt.) Typischerweise kennzeichnet man ihn durch eine Zahl – der Spin ist immer ein Vielfaches von 1/2 (oder Null) in passend gewählten physikalischen Einheiten, es gibt also Spin-1-Teilchen oder Spin-3/2-Teilchen.

Schauen wir erst mal wieder auf ein Photon. Aus Photonen kann man ja eine elektromagnetische Welle bauen, so wie die hier:

emWave

In grün seht ihr das elektrische Feld, in magenta das Magnetfeld, der gelbe Pfeil kennzeichnet die Ausbreitungsrichtung. Um diese Welle so zu rotieren, dass sie genauso aussieht wie vorher, müsst ihr sie entlang der gelben Achse um eine volle 360°-Drehung drehen. Daraus folgt, dass der Spin des zugehörigen teilchens gleich 1 sein muss. (Ein Spin-0-Teilchen hätte eine Symmetrie wie eine Kugel – es sieht immer gleich aus, egal wie ihr es dreht. Es kann auch Spin-0-Wellen geben, beispielsweise eine Dichtewelle. Teilchen mit Spin 1/2 müsst ihr um 720° drehen, bis der Zustand wieder derselbe ist – das klingt verrückt, ist aber tatsächlich so.)

Das Graviton kann allerdings kein Spin-1-Teilchen sein, denn wenn Spin-1-Teilchen eine Kraft übertragen, dann ist die Kraft zwischen Teilchen gleicher Ladung abstoßend, so wie bei der elektrischen Ladung. Wir wissen aber, dass alle Massen sich anziehen, auch wenn wir zwei identische Teilchen betrachten. Der Spin eines Gravitons muss deshalb ein Vielfaches von 2 sein. (diesen Zusammenhang zwischen Spin und Anziehung/abstoßung zu erklären, ist wirklich knifflig – meinen Versuch findet ihr hier, aber um den Artikel zu verstehen, müsst ihr euch vermutlich durch die gesamte Artikelserie durchackern.)

Am einfachsten kann man sich den Spin des Gravitons ähnlich übrlegen wie eben beim Photon: Wir schauen uns eine Gravitationswelle an und gucken, wie man sie drehen muss, damit das Bild sich nicht ändert. So verzerrt eine Gravitationswelle den Raum (Bild von Einstein online , dank an Markus Pössel):

cyl_slice

Ihr seht, dass das Bild zu jedem Zeitpunkt so ist, dass es sich bei einer Drehung um 180° nicht ändert. Daraus folgt direkt, dass das Graviton ein teilchen mit Spin 2 sein muss.

1 / 2 / 3 / 4 / 5 / 6 / 7 / 8

Kommentare (129)

  1. #1 MX
    19. April 2015

    Beim Versuch, die Sache mit der Quantengravitation zu verstehen, habe ich bis zum PS glatt überlesen, dass du nur weibliche Formen verwendest. Was immer das bedeutet. Traust du dir anhand des Standes der Dinge eine Prognose zu, wie lange es dauert, bis das Problem geknackt ist? Eher 10 Jahre? Eher 50 Jahre?

  2. #2 A-P-O
    19. April 2015

    Inwiefern werfen ST und Schleifen-QT jede Menge Unsinn aus?

  3. #3 MartinB
    19. April 2015

    @MX
    Nein, das Problem ist seit Ewigkeiten bekannt, und obwohl es ja viele Ideen gibt, ist man nicht wirklich weiter. Ich glaube, irgendwo braucht man was fundamental neues in Sachen Beschreibung der Raumzeit, habe aber keine Idee, was das sein soll.

    @A-P-O
    Ich bin für beide kein Experte – in der ST ist die QG ja nach wie vor eine Spin-2-Theorie, und deswegen sollte man da doch dieselben Renormierungsprobleme haben. Loop-QG kenne ich zu wenig, um das im einzelnen beurteilen zu können; aber fertige Theorien der QG sind sie beide nicht; sonst hätten wir ja eine solche Theorie.

  4. #4 Turi
    19. April 2015

    Das Ganze klingt ein wenig wie das Problem der Ultraviolettkatastrophe der klassischen Physik. Und dort hat die Auflösung des Problems eine ganzen neuen Physikzweig geschaffen.
    Was alles noch viel spannender macht.

  5. #5 hugo
    20. April 2015

    Ist nicht der Lense-Thirring-Effekt ein gravitomagnetischer Effekt? Dann müsste doch Gravity Probe B als Nachweis des Gravitomagnetismus gelten.

  6. #6 Reggid
    20. April 2015

    mir wurde mal in einem seminar beigebracht, dass man jedem, der behauptet, dass quantentheorie und gravitation nicht vereinbar sind, sofort widersprechen müsse um klarzustellen, dass diese aussage (wenn auch oft wiederholt), so nicht richtig ist.

    gravitation lässt sich ganz normal ohne besondere schwierigkeiten im rahmen einer effektiven quantenfeldtheorie darstellen. diese hat natürlich einen beschränkten gültigkeitsbereich (unterhalb der planck-skala), darin unterscheidet sie sich aber eigentlich nicht von den anderen grundkräften (wenn man davon ausgeht dass das standardmodell der elementarteilchenphysik ja auch keine fundamentale theorie ist und heute ja auch schon als effektive feldtheorie behandelt wird).

    das ändert natürlich nichts daran, dass wir auch damit keine physik an der planck-skala beschreiben können, aber dass eine quantenfeldtheorien bei beliebig hohen energien nicht mehr konsistent ist kennt man auch aus der QED (landau pol). trotzdem behauptet niemand, dass elektromagnetismus nicht mit der quantentheorie vereinbar wäre. natürlich ist die QED keine theorie die wir ohne weiters bis zu unendlich hohen energien extrapolieren können, und dasselbe gilt auch für die effektive feldtheorie der quantengravitation.

    also eine quantentheorie der gravitation (also eine vereinheitlichung von ART und quantenfeldtheorie) ist möglich und wir kennen sie bereits. nur ist die in ihrem gültigkeitsbereich beschränkt (so wie jedes andere physikalische modell auch), und liefert insbesondere keine vorhersagen für die physik an der planck-skala. und darin wäre man ja eigentlich interessiert, wenn man besser verstehen will was in schwarzen löchern passiert oder wenn man näher an den urknall extrapolieren will. aber eine konsistente, funktionierende quantentheorie der gravitation ist es trotzdem.

    https://arxiv.org/pdf/1209.3511v1.pdf

  7. #7 MartinB
    20. April 2015

    @Hugo
    Stimmt, danke für den Hinweis. Habe mal einen Satz im Artikel ergänzt.

    @Reggid
    Aber im Gegensatz zur QED ist die QG bereits auf zwei-loop-level nicht mehr renormierbar, so dass man nicht mal einen Propagator korrekt definieren kann; ich denke, das ist schon ein gewichtiger Unterschied.
    Na klar kann eine nicht renormierbare Theorie trotzdem in Grenzen sinnvoll und korrekt sein, das galt ja auch für die Fermi-Theorie der schwachen Wewi.
    Danke auch für den Link, schaue ich mir bei Gelegenheit an.

  8. #8 a.n
    20. April 2015

    @Turi

    Ja genau das ist es. Und genauso, wie die Quantelung der Wirkung das UV-Problem der Schwarzkörperstrahlung aufgelöst hat, könnte eine “Quantelung der Raumzeit” UV-Divergenzen in Quantenfeldtheorien auflösen. Entsprechende Ansätze gibt es in der LQG.

  9. #9 JoJo
    20. April 2015

    @MartinB

    Teilchen mit Spin 1/2 müsst ihr um 720° drehen, bis der Zustand wieder derselbe ist – das klingt verrückt, ist aber tatsächlich so.

    Dazu gibt es einen schönen Versuch:

    Man stellt sich eine gefüllte Tasse auf die Handfläche und versucht — ohne die Füße von der Stelle zu bewegen — die Tasse um ihre eigene (vertikale) Achse zu drehen, und zwar ohne Flüssigkeit zu vergießen und so, daß die Tasse ihre Position relativ zur Handfläche nicht ändert.

    Wer ein bischen beweglich ist, schafft es problemlos, die Tasse um 360° zu drehen — Arm und Schulter sind dann allerdings in einer unkomfortablen Stellung und nicht in der Ausgangsposition. Man kann sogar weiter drehen bis zu einem Winkel von 720°, und der Arm ist dann wieder in der Ausgangsposition!

    Das ganze geht natürlich auch ohne Tasse, indem man die Handfläche immer schön nach oben zeigen lässt.

  10. #10 sffd
    20. April 2015

    Die ST ist die einzige konsistente QG Theorie. Lern doch mal endlich was darüber Martin anstatt immer nur Fragen danach zu stellen, es gibt doch gute Einführungen z.B. von David Tong.

  11. #11 sffd
    20. April 2015

    Für die Grundlagen braucht man auch nicht holomonie/homotopie Theorie und ähnliche Sachen. ART und QFT reicht. Becker/Schwarz ist zum Beispiel sehr gut.

  12. #12 sffd
    20. April 2015

    holonomie*

  13. #13 Reggid
    21. April 2015

    @MartinB

    renormierbarkeit im “altmodischen” sinne (so wie wir sie z.B. in der QED tatsächlich haben) ist im rahmen von effektiven feldtheorien ohnehin nicht gefordert. also unterscheidet sich eine effektive feldtheorie der quantengravitation in diesem punkt auch nicht wirklich von anderen.

  14. #14 Artur57
    21. April 2015

    Mal Frage: was zwingt einen denn zu der nobelpreisverhindernden Annahme, dass sich die Gravitonen aufspalten und somit eine Selbstbeeinflussung der Gravitation gegeben ist? Ein Experiment kann es ja wohl nicht sein, das Graviton ist ja nicht einmal nachgewiesen. Auch spricht die Beobachtung dagegen: ein Schwarzes Loch kann einen Lichtstrahl zwar zur Umkehr zwingen, die Gravitation selbst aber verlässt das Loch völlig ungehindert. Was ja eigentlich erstaunlich ist. Und auch: die Gravitation nimmt mit dem Abstandsquadrat ab, darauf ist immer Verlass und es gab bislang keine Ausnahme.

    Warum also diese Annnahme einer “Selbstbeeinflussung”

  15. #15 MartinB
    21. April 2015

    @sffd
    Wenn ich mal zeit habe, schaue ich mir auch die Stringtheorie an.
    Gibt es eigentlich eine ST-Berechnung des Graviton-Propagators auf n-Loop-Level? (siehe auch die Frage unten)

    Ansonsten warte ich darauf, dass sie mal eine Vorhersage macht, die nicht unseren Beobachtungen widerspricht und dann wieder mühsam wegdiskutiert werden muss (Extra-Dimensionen, Supersymmetrie etc.).

    @Reggid
    Vielleicht verstehe ich da auch was falsch, aber auch wenn ich bei irgendeiner Planck-Skala abschneide sollten doch die n-Loop-Korrekturen für einen Graviton-Propagator ein ziemliches Problem machen, oder sehe ich das falsch

  16. #16 MartinB
    21. April 2015

    @Artur57
    Einfach auf den Punkt gebracht: Die Theorie ist 1. inkonsistent und 2. nicht im Einklang mit der ART, wenn wir annehmen, dass die Energie, die in einem Gravitationsfeld steckt, selbst keine Quelle von Gravitationskräften ist.

  17. #17 MartinB
    21. April 2015

    PS: Schwarze Löcher würde es dann auch nicht geben, soweit ich sehe.

  18. #18 Cobi
    21. April 2015

    @MartinB

    Higher loop Rechnungen in String Theory sind ziemlich komplex und ich kenne mich nicht besonders gut damit aus, aber im nLab wird der Stand der Dinge ganz gut zusammen gefasst:

    https://ncatlab.org/nlab/show/string+scattering+amplitude#References

    Ich war vor zwei Wochen bei einer String Schule wo Amplituden tatsächlich ein sehr großes Thema waren. Es sind was das angeht also wirklich noch viele Fragen offen.
    Zumindest für die UV finitenes gibt es aber anscheinend sehr gute Argumente.
    Eine technische Vorhersage der String Theorie ist beispielsweise AdS/CFT Dualität, welche es ermöglicht Quantengravitation in D Dimensionen über eine dualen Eichtheorie in D-1 Dimensionen zu untersuchen, welche identische Physik liefern sollte.

    Interessant sind vielleicht auch in der sechsten Thong lecture
    https://www.damtp.cam.ac.uk/user/tong/string/six.pdf
    die Abschnitte “High Energy Scattering” und “Graviton Scattering”, (die Rechnungen beziehen sich auf bosonische Strings).

  19. #19 Cobi
    21. April 2015

    Das sollte natürlich Tong heißen. 😉

  20. #20 MartinB
    21. April 2015

    @Cobi
    Danke für die Infos, der zweite Link sieht echt so aus, als könnte ich denmit etwas Mühe verstehen, mal gucken.

    “Zumindest für die UV finitenes gibt es aber anscheinend sehr gute Argumente.”
    Meine Überlegung/Frage dazu ist diese hier, vielleicht kannst du ja schnell was dazu sagen (ich hoffe, ich drücke mich nicht zu unsauber aus):
    So ein 2-loop-Diagram wie meins oben ist nicht renormierbar. Jetzt kann man sagen, macht ja nix, ich betrachte die Theorie als effektive Theorie und schneide bei der Planck-Skala ab, weil da die strings dafür sorgen, dass nichts divergiert.
    Dann bekomme ich aber immer noch eine Korrektur an meinen Propagator (und entsprechend für jedes andere Diagramm) – die ist endlich, so wie bei normalen Renormierungsrechnungen auch, wo man erst nen endlichen cutoff hat und den dann am Ende kontrolliert nach unendlich schickt.
    Diese endliche Korrektur sollte doch fürs Graviton ziemlich groß sein (weil wir auf der Planckskala dann ja effektiv ne große Kopplungskonstante haben) – müssten diese endlichen Korrekturen, die sich ja für jede höhere Ordnung ergeben, am Ende einen Effekt haben? Kann man den berechnen?

  21. #21 Cobi
    21. April 2015

    @MartinB

    Das scheinbare Problem entsteht dadurch, dass es sich bei der Low energy effective action welche man erhält indem man String Amplituden ausrechnet nicht um eine Wilsonian effective action handelt, sondern um eine Quantum effective action.
    Der Unterschied wird beispielsweise hier
    https://www.rug.nl/research/portal/files/2939301/thesis.pdf
    auf Seite 33 beschrieben.
    Grob gesprochen geht es um folgendes:
    Bei einer Wilsonian effective action nimmt man an, dass einige schwere Teilchen, welche in Loops auftauchen würden, ausintegriert wurden und daher die Theorie renormiert werden muss.
    Wenn ich jetzt aber eine String Amplitude berechne, habe ich in den loops nicht nur die schweren, sondern alle Teilchen berücksichtigt. Die entstehende Low energy effective action ergibt also nur “klassisch” d.h. auf tree-level Sinn, enthält aber Quanten-Effekte bis zur loop Ordnung welche ich in der String Rechnung berücksichtigt habe.
    Da man also gar keine Loops mehr berechnet, stellt sich auch die Frage nach dem Cut-off oder daraus resultierenden Korrekturen nicht mehr. 🙂

  22. #22 Cobi
    21. April 2015

    Vielleicht noch ein Zitat aus der siebten lecture von David Tong zur Motivation für Strings:
    “Something rather remarkable has happened here. We started, long ago, by looking at how a single string moves in flat space. Yet, on grounds of consistency alone, we’re led to the action (7.16) governing how spacetime and other fields fluctuate in D = 26 dimensions. It feels like the tail just wagged the dog. That tiny string is seriously highmaintenance: its requirements are so stringent that they govern the way the whole universe moves.”

  23. #23 MartinB
    21. April 2015

    @Cobi
    Danke, aber den Kommentar in #21 habe ich nicht verstanden, da fehlt es anscheinend doch an ein bisschen grundwissen. Ich dachte, bei den Stringberechnungen muss man auch loops (bzw. unterschiedliche Topologien der Weltflächen) berechnen.

  24. #24 Cobi
    21. April 2015

    @MartinB
    Ja, mein letzter Satz war sehr missverständlich. Die Loops muss man nur in der Stringtheorie berechnen, die entstehende “Low energy effective action” berücksichtigt diese Loop Effekte dann im QFT Tree-level. Die String loop Rechnungen sind für n>1 zwar wahnsinnig kompliziert aber grundsätzlich berechenbar (einige Rechnungen wurden wenn ich mich richtig an den Vortrag von Berkovits erinnere bis 4- oder 5-loop durchgeführt) und es gibt Grund zu der Annahme, dass sie für alle n ein endliches ergebnis liefern.

  25. #25 Reggid
    21. April 2015

    natürlich können auch in einer quantum gravity effective theory loops berechnet werden, und diese müssen auch renormiert werden (was aber zuminest ordnung für ordnung problemlos möglich ist).

    siehe z.B hier kapitel 7: https://arxiv.org/pdf/gr-qc/9512024v1.pdf

  26. #26 MartinB
    22. April 2015

    @Cobi&Reggid
    Danke erstmal. Mal sehen, wann ich dazu komme, mich da was anzulesen.

  27. #27 JoJo
    22. April 2015

    Wenn man die elektrische Anziehung zwischen zwei Elektronen vergleicht mit deren Anziehung aufgrund der Gravitation, ergibt sich sich ein Faktor von ca. -4·10^42.

    Der exakte Wert dieser Konstanten müsste sich doch aus einer vereinheitlichten Theorie ergeben?

    Ist die Größe der Konstanten ein Hinweis darauf, dass zwei grundsätzlich verschiedene Mechanismen am Werk sind? 10^42 ist ja nicht gerade wenig…

  28. #28 MartinB
    23. April 2015

    @JoJo
    Ja, das ist eine gute Frage, das sogenannte Hierarchieproblem.
    https://de.wikipedia.org/wiki/Hierarchieproblem
    (der Artikel ist aber nicht sooo gut verständlich, finde ich, die englische Version ist anscheinend etwas besser)
    Ne gute Antwort darauf hat niemand.

  29. #29 Harleaquin
    23. April 2015

    Könnte vielleicht ein “Streben zu Flachheit” des Raumes das Renormierungsproblem lösen?
    Quasi eine Spannkraft, die das erscheinen virtueller Gravitonen mit hohem Impuls/Energie und somit großer lokaler Krümmung eindämmt.
    Geschickt austariert könnte das selbe “Streben nach Flachheit” auch die Ausdehnung des Raum selbst erklären.

  30. #30 MartinB
    23. April 2015

    @Harleaqin
    So ein streben ist in den Gleichungen schon drin – höher gekrümmte Räume haben ja auch eine höhere Energie (das steckt letztlich schon in der Energieformel E=h f= hc/Wellenlänge drin).
    Man müsste einen zusätzlichen Term einbauen, aber ohne dass der den rest der Theorie kaputt macht. (Und wenn man bedenkt, dass die Frage, ob eine Raumkrümmung stark oder schwach ist, von der Beobachterin abhängt, wird das evtl. schon knifflig – denn lokal gibt es immer ein “freier-Fall”-Bezugssystem, in dem der Raum flach aussieht.)
    Kurz: ich sehe nicht, dass das geht, will aber nicht ausschließen, dass man das mit hinreichend trickreichen Methoden hinbekommt

  31. #31 Harleaquin
    23. April 2015

    Stimmt, daß Raumkrümmung beobachtungsabhängig ist hatte ich nicht auf meinem (naiven) Schirm. Da geht sie dahin, die schöne Idee 🙂

    Danke jedenfalls für die prompte Antwort und den interessanten Artikel, der mich schon zwei Tage zum Grübeln anregt.

  32. #32 MartinB
    23. April 2015

    @Harleaquin
    “der mich schon zwei Tage zum Grübeln anregt.”
    Das ist doch gut.

  33. #33 Quantom
    24. April 2015

    Vielen Dank für diesen Artikel!
    Es hat mich zum besseren Verständnis über die Quantengravitation beigetragen.

    Super , weiter so!!

  34. #34 MartinB
    24. April 2015

    @Quantom
    Freut mich.

  35. #35 Lucia HY
    Deutschland
    25. April 2015

    Das Bild der elektromagnetischen Welle zeigt eine stehende Welle, keine propagierende. Bei einer propagierenden Welle sind E-Feld und B-Feld um 90° verschoben, Sinus und Kosinus (Der Nabla-Operator der Maxwellgleichung ist eine erste Ableitung nach den Ortskoordinaten). Ansonsten gäbe es während der Schwingung Zeitpunkte, wo die Energie Null ist, die Energie oszilliert aber zwischen E-Feld und B-Feld hin und her: Dort, wo das E-Feld ein Maximum hat, ist das B-Feld gerade Null und umgekehrt.

  36. #36 rolak
    25. April 2015

    um 90° verschoben

    Fern/Nahfeld verwechselt, Lucia HY?

  37. #37 MartinB
    25. April 2015

    @Lucia
    Ist es nicht genau andersrum – Phasenverschiebung bei der stehenden Welle?

  38. #38 Physik-Fan
    22. Mai 2015

    @ sffd
    > Die ST ist die einzige konsistente QG Theorie.

    Es geht zwar hier um die QG (darauf komme ich noch), aber es stellt sich zunächst die Frage, nach der Substanz des ST-theoretischen Ansatzes. Seit mehr als 30 Jahren modelliert und rechnet man damit herum, aber eine experimentell verifizierbare spezifische Vorhersage gibt es bis heute nicht. Von einer Bestätigung ganz zu schweigen. Das Fehlen einer solchen Vorhersage kritisiert übrigens Lisa Randall (Zitate könnte ich nennen). Dagegen kann man einwenden, dass die experimentellen Möglichkeiten für die ST nicht ausreichen. Man könnte dann gleich wieder Randall zitieren, die so ein Programm aber nicht für aussichtsreich hält, erst recht nicht, wenn über den riesigen Energiebereich bis zur Planck-Energie hinweg extrapoliert wird. Sie glaubt nicht, dass man so die richtige Theorie treffen kann und hält den iterativen Weg im Wechselspiel zwischen Theorie und Experiment für zielführend (sie nennt das Modellbau).

    Es wird oft die Ansicht geäußert, die ST wäre nicht falsifizierbar. Das mag prinzipiell so sein, aber ihre Glaubwürdigkeit ist durchaus erschütterbar und zwar konkret. Die Crux der ST ist die SUSY, über die ja die Fermionen reinkommen. Die SUSY ist jedoch bereits angezählt. Sie hätte eigentl. schon im LHC auftreten sollen und zwar um 250 GeV, dem Energieniveau der Schwachen WW. Zeigt sich im LHC bis 1 TeV wieder nichts (derzeit laufen die ersten Kollisionen mit 13 TeV, es wird spannend!), ist die SUSY strenggenommen nicht widerlegt, schließlich kann die Symmetriebrechung (übrigens eine reine Ad-Hoc-Hypothese, man hat keinen Mechanismus dafür) höher getrieben werden, aber ihr Stern wird sinken. Zumal sie ihre positiven Effekte, nämlich die Glättung des Vakuums und Konvergenz der WW’en, bis dahin längst verloren hat. Die Auswirkung auf die ST kann man an drei Fingern abzählen oder hat man sie schon gegen “SUSY-Ausfall” gehärtet?

    Gravierender als die experimentellen Aspekte sind für mich jedoch die prinzipiellen Eigenschaften des ST-Ansatzes. Es ist ein Konglomerat von Hypothesen, übereinander gestapelt. Zählt man sie alle auf, je nach Breite der Betrachtung, kommt man auf zig davon. Wie kann meinen, dass das alles zutrifft? Das ist mir ein Rätsel. Theorien sollten auf wenigen Hypothesen beruhen, um erfolgsversprechend zu sein. Musterbeispiele sind SRT (ist sogar hypothesen-frei) und ART. Es gibt z.B. null Evidenz für die Existenz höherer Dimensionen, sogar entgegensprechende Indizien (Doppel-Pulsar-Systeme). Die ins Feld geführte “Schönheit” der Mathematik, ist kein stichhaltiges Argument für Physik. Darauf weißt übrigens auch Randall hin. Man könnte grob sagen, Mathematik ist nicht Physik. Das hat man vielleicht schon vergessen.

    Nun zu QG. Die ST enthält zwar das Graviton, aber ist sie konform zur ART (genauer gesagt ihrer höherdimensionalen Erweiterung)? Dazu müsste die Raumzeitgeometrie in der ST variabel sein. Ihre Gleichungen sind aber nicht diffeomorphismus-invariant und sie ist auf einer vorgegebenen Raumzeit definiert. Das ist jedenfalls der mir bekannte Stand.

  39. #39 Niels
    23. Mai 2015

    @Physik-Fan

    Musterbeispiele sind SRT (ist sogar hypothesen-frei)

    Wie meinen? Sind die Einsteinschen Postulate für dich etwas anderes?

    Gravierender als die experimentellen Aspekte sind für mich jedoch die prinzipiellen Eigenschaften des ST-Ansatzes. Es ist ein Konglomerat von Hypothesen, übereinander gestapelt. Zählt man sie alle auf, je nach Breite der Betrachtung, kommt man auf zig davon.

    Kennst du dich da aus? Könntest du diese Postulate/Hypothesen dann mal kurz aufzählen?
    Ich hab dazu nämlich noch nirgends eine anständige Zusammenfassung gesehen, das würde mich wirklich interessieren.

  40. #40 MartinB
    23. Mai 2015

    @Physik-Fan
    Schließe mich Niels an, das würde ich auch gern mal wissen.

  41. #41 Physik-Fan
    26. Mai 2015

    @Niels

    > Musterbeispiele sind SRT (ist sogar hypothesen-frei)

    > Wie meinen? Sind die Einsteinschen Postulate für dich etwas anderes?

    Es gibt gute Literatur zur Entwicklung und Ideengeschichte der RT, z.B. “Auf den Schultern von Riesen und Zwergen” von Renn (wohl der beste Einstein-Kenner) oder die umfangreiche Einstein-Biographie von Fölsing.

    Einstein hat selbst die SRT als “Relativitätsprinzip” bezeichnet und erst später die Bezeichnung der anderen als “…theorie” übernommen. Tatsächlich hat die SRT einen prinzipiellen Charakter. Sie beruht auf dem Relativitätsprinzip, welches selbst wieder auf dem Galileischen Prinzip (dem Relativitätsprinzip der klass. Mechanik) und der LG-Invarianz beruht. Das Galileische Prinzip ist weitgehend evident und bildet einen Rahmen für die Physik in gleichförmig bewegten Bezugssystemen. Das ist der Unterschied zu einer Hypothese, die mehr eine spezifische Annahme ist. Es würde auch ziemlich komisch klingen, das Galileische Prinzip als “Galileische Hypothese” zu bezeichnen.

    Die LG-Invarianz ist zunächst ein empirischer Befund, wobei Einstein bestritten hat, vom Michelson-Morley-Experiment gewusst zu haben und es theoretisch aus den Maxwellschen Gleichungen geschlossen hat, in denen es keine stehenden elektromagn. Wellen gibt.

    Die SRT ist also frei von Hypothesen, anders wie die ART. Die Krümmung der Raumzeit durch Materie ist eine Hypothese und für mich auch die Geodäte als Raumzeitbahn.

  42. #42 Physik-Fan
    26. Mai 2015

    @Niels

    > Gravierender als die experimentellen Aspekte sind für mich jedoch die prinzipiellen
    > Eigenschaften des ST-Ansatzes. Es ist ein Konglomerat von Hypothesen, übereinander
    > gestapelt. Zählt man sie alle auf, je nach Breite der Betrachtung, kommt man auf zig davon.

    > Kennst du dich da aus? Könntest du diese Postulate/Hypothesen dann mal kurz aufzählen?
    > Ich hab dazu nämlich noch nirgends eine anständige Zusammenfassung gesehen, das
    > würde mich wirklich interessieren.

    Eine Zusammenstellung habe ich nicht, aber hier schon einige Male was dazu geschrieben. Es ist eigentl. durch das Konzept des Strings schon klar, dass dessen Komplexität und Variabilität zu Hypothesen führen wird, mehr jedenfalls als für ein punktförmiges Teilchen.

    Hier einige Hypothesen aus meiner Sicht:

    – Der Strings selbst. Dabei ergibt sich gleich die Frage, was er überhaupt ist. Frei schwingende Energie, losgelöst von einem Träger, was soll das?
    – Wie der String sich in der Raumzeit bewegt, wie er die Teilcheneigenschaften hervorbringt, wie er wechselwirkt. Das mag i.W. auf einfachen Annahmen beruhen, aber es sind Annahmen.
    – diverse String-Typen und Branen-Typen, alle frei erfunden
    – Die 10- bzw. 11-dimensionale Raumzeit. Man kann das ja recht gut als notwendig für die ST gezeigt haben, aber dann hängt es an der Theorie, ohne eine eigenständige Begründung dafür. So eine ungeheuer weitgehende und dazu spezielle Erweiterung der Raumzeit rein theoretisch zu begründen, ist mehr als gewagt. Existieren höhere Dimensionen überhaupt? Warum sollte die Raumzeit exakt so sein? Nur damit die ST funktioniert? Bisher haben sich jedenfalls bei Feinmessungen der Grav. (Doppel-Pulsar-System) keine Abweichungen von der ART ergeben. Lüst erwähnt, dass sie über eine Formulierung der ST auf der 4-D-Raumzeit nachdenken, mit abstrakten Extradimensionen, etwa nach Art der fermionischen in der SUSY. Es ginge also evtl. auch anders. Ein generell fragwürdiger Punkt ist für mich auch das damit verbundene Konzept der dimensionalen Einbettung. Ist das physikalisch sinnvoll?
    – 6 (bzw. 7) räuml. Dimensionen sind kompaktifiziert. Warum sind sie kompaktifiziert und die restlichen ausgedehnt?
    – Die kompaktifizierten Dimensionen bilden einen Calabi-Yau-Raum. Wieder die Frage warum? Es könnte doch auch ein anderer sein. Bei Lüst kann man lesen, dass sie mal über Alternativen nachdachten und Gross (ich glaube, der war’s) dazu fragten. Er riet davon ab und sie ließen es. Komische Art, in Physik zu forschen …
    – Die Gültigkeit der SUSY wird vorausgesetzt. Sie beruht ihrerseits auf Hypothesen und ist ohne ein empirisches Indiz. Sogar indirekte blieben bisher aus, anders als es z.B. beim Higgs-Mechanismus war. Was ist, wenn im LHC wieder keine SUSY-Teilchen auftauchen?

    – Die String Cosmology, etwas noch spekulativeres. Wieder wird ein String-Typ erfunden, der Cosmic Superstring. Es ergibt sich ein Ansatz, warum 3 Raumdimensionen ausgebreitet sind. Wäre eine feine Sache, aber leider steckt man in ungelösten Schwierigkeiten für die Calabi-Yau-Kompaktifizierung, Für Branen ist es noch problematischer.

    Das habe so mal zusammengeschrieben. Jetzt müsste man es erweitern und verfeinern.

    ( die Einrückkung dürfte z.T nicht passen, hab’s nicht besser hinbekommen)

  43. #43 MartinB
    26. Mai 2015

    @Physik-Fan
    “Die SRT ist also frei von Hypothesen, anders wie die ART”
    Hmm, wenn man empirische Erkenntnisse (Galilei-Invarianz zähle ich durchaus dazu, denn rein ohne jede Empirie könnte Invarianz auch für Drehbewegungen gelten, was es nicht tut) reinsteckt, dann ist eine Theorie doch nicht mehr hypothesenfrei, oder? Oder haben wir unterschiedliche Vorstelungen, was eine “Hypothese” ist – sollte man hier nicht eher von einem Postulat o.ä. sprechen, weil es etwas ist, was an der Basis der Theorie steht.

    Was deine Kritik an der Stringtheorie angeht, kann ich die nur zum Teil nachvollziehen – auch in der normalen QFT wissen wir nicht, was ein Quantenfeld “ist” und was da “schwingt”.

  44. #44 Niels
    27. Mai 2015

    @Physik-Fan
    Erst einmal etwas grundlegendes:
    Mir ist nicht ganz klar, was du unter dem Begriff Hypothese verstehst. Könntest du Hypothese mal definieren? Was ist für dich ein Postulat?

    Zur SRT

    Tatsächlich hat die SRT einen prinzipiellen Charakter. Sie beruht auf dem Relativitätsprinzip

    Das ist bei der ART doch genauso, wobei deren allgemeines Relativitätsprinzip sogar ein weniger spezielles Postulat und damit meiner Meinung nach “prinzipieller” ist.

    Die SRT ist also frei von Hypothesen, anders wie die ART. Die Krümmung der Raumzeit durch Materie ist eine Hypothese und für mich auch die Geodäte als Raumzeitbahn.

    Die Krümmung der Raumzeit und Geodäten als Weltlinien folgen aber doch aus den Postulaten der ART.
    Sie sind also in dem selben Sinne “Hypothesen” der ART wie es Zeitdilatation und relativistische Massenzunahme für die SRT sind, oder?

    Zu deiner Kritik an den “Hypothesen” der Stringtheorie:
    Kann ich wie MartinB auch nicht ganz nachvollziehen. Ich hab mir ehrlich gesagt auch eine etwas fachspezifischere Antwort erhofft, etwa im Stil der Postulate der Quantenmechanik.
    Dann hätte man vergleichen können, ob die Stringtheorie wesentlich mehr Postulate nötig hat.
    (Wie MartinB schon erwähnt hat, bei den QFTs wirds nicht besser.)

    Der Strings selbst. Dabei ergibt sich gleich die Frage, was er überhaupt ist.
    Frei schwingende Energie, losgelöst von einem Träger, was soll das?

    Etwas mathematisch Definiertes (wobei Messvorschriften und Messgrößen eingehen), so wie es Objekte einer grundlegenden physikalischen Theorie immer sein muss.

    Als Beispiel:
    Was ist überhaupt ein Elementarteilchen?
    Eine bestimmte Anregungsstufe eines bestimmten Feldes, was soll das?

    Wie der String sich in der Raumzeit bewegt, wie er die Teilcheneigenschaften hervorbringt, wie er wechselwirkt. Das mag i.W. auf einfachen Annahmen beruhen, aber es sind Annahmen.

    Solche Annahmen sind doch ebenfalls in jeder grundlegenden physikalischen Theorie zwingend notwendig.

    diverse String-Typen und Branen-Typen, alle frei erfunden

    Diverse Quantenfelder, alle frei erfunden.

    Die 10- bzw. 11-dimensionale Raumzeit. Man kann das ja recht gut als notwendig für die ST gezeigt haben, aber dann hängt es an der Theorie, ohne eine eigenständige Begründung dafür.
    […]Die kompaktifizierten Dimensionen bilden einen Calabi-Yau-Raum. Wieder die Frage warum?

    Die pseudo-riemannsche Mannigfaltigkeiten der ART, die Hilberträume der Quantenmechanik, …
    Warum? Weil es funktioniert?

    Bei diesen Punkten sehe ich daher eigentlich keine grundlegenden Probleme.
    In der Mathematik der Stringtheorie kenne ich mich zwar überhaupt nicht aus, dass eine Quantengravitation allerdings mathematisch äußerst anspruchsvoll sein wird und viele neuartige Konzepte einführen muss, halte ich für ziemlich sicher.

    Etwas anders ist die von dir gestreifte mangelnde Falsifizierbarkeit der Stringtheorie.

  45. #45 MartinB
    27. Mai 2015

    @Niels
    Ich habe gerade eine vollständige Kopie von Hatfields QFT for point particles&strings gefunden – erst mal, um noch mal normale QFT zu verstehen, aber ich hoffe, da bekomme ich zumindest einen kleinen Einblick in die Stringtheorie. Mal sehen, vielleicht kann ich ja mal irgendwann was drüber bloggen…

  46. #46 Niels
    28. Mai 2015

    Würde mich freuen. Immer frisch ans Werk. 😉
    Weitere QFT-Artikel sind auch sehr willkommen.

  47. #47 MartinB
    28. Mai 2015

    Ja, ich beiße mich leider gerade an den grundlagen fest, weil ich nicht verstehe, was genau ein Erzeugungs-/Vernichtungsoperator physikalisch bedeuten soll.

  48. #48 Physik-Fan
    30. Mai 2015

    @MartinB
    @Niels

    Wir kommen da in tiefes Fahrwasser und ich bezweifele fast, dass ein Blog das geeignete Medium für eine solche Diskussion ist, zumindest nicht dieser Thread. Wäre durchaus interessant diese Metaebene in einem eigenen Thema zu behandeln. Vielleicht gibt’ s das auch schon irgendwo in ScienceBlogs.

    Was ist eine Hypothese? Ich sag’s mal in eigenen Worten: Es handelt sich um eine Aussage, für die man evtl. gewisse Argumente bringen kann, die aber nicht belegt ist. Ein experimenteller Befund ist keine Hypothese, sondern eine Tatsache. Bspwse. galt die LG-Invarianz nach dem Michelson-Morley-Experiment als nachgewiesen.

    Interessant ist die Frage, ob eine notwendige Aussage auch als Hypothese betrachtet werden kann, da die Aussage ja aus Prämissen der Theorie ableitbar ist. Ich bin der Meinung, dass man auch hier von einer Hypothese sprechen kann, da die Aussage einen allgemeineren Charakter hat und ihre Negation die Prämissen negiert. Die Aussage sollte eigens begründet werden, vor allem, wenn es eine weitreichende Aussage ist. Eine solche ist z.B. die 10- bzw. 11-dimensionale Raumzeit der ST. Da ist doch zunächst mal die Frage, ob es überhaupt Belege für die Existenz höherer Dimensionen gibt. Tatsache ist, dass es bisher keine gibt, keine Abweichung von der ART, selbst bei den feinsten Messungen (Doppel-Pulsar-System) oder ausbleibende Mini-SLs im LHC. Natürlich kann man sagen, dass die Effekte für die Experimente zu klein oder hochenergetisch sind, aber es ist nun man so, dass sich nichts gezeigt hat, obwohl das schon der Fall hätte sein können.

    Ein Postulat ist lt. Wikipedia eine grundlegende Aussage für eine Theorie oder ein formales System, die nicht aus Gegebenem abgeleitet werden kann. Etwa die Axiome der euklidischen Geometrie sind Postulate. Sie sind nicht zu begründen ist, sondern werden vorausgesetzt. Im Unterschied dazu möchte man Hypothesen belegen, ihre Gültigkeit aufzeigen. So gesehen sind Hypothesen mehr was für die Physik, Postulate mehr was für formale Systeme.

    Ein Prinzip in der Physik würde ich als eine Aussage bezeichnen, deren Gültigkeit evident ist (in einem bestimmten Zusammenhang) und die einen Rahmencharakter für die Physik (in diesem Zusammenhang) hat. Beispiel sind Galileiprinzip, Relativitätsprinzip, Äquivalenzprinzip.

    Die SRT beruht auf dem Relativitätsprinzip. Alle ihre Effekte sind streng logisch ableitbar, deswegen hat sie den Charakter eines Prinzips. Einstein bezeichnete auch den ganzen Komplex als “Relativitätsprinzip”. Bei der ART habe ich mal nachgedacht, was Raumzeitkrümmung und Feldgleichung sind, vielleicht doch mehr begründet bzw. abgeleitet, als hypothetisch. Die Plausibilität der Raumzeitkrümmung hat Einstein in einer Reihe von Gedankenversuchen gut herausgearbeitet. Die Feldgleichung ist bei ihm aber eine Hypothese. Sie wurde nicht abgeleitet ist, sondern mit ein paar Kriterien (Kovarianz, newtonscher Grenzfall etc.) erraten. Später konnte sie Cartan wohl herleiten, aber damit habe ich mich noch nicht näher beschäftigt. Die riemannsche Geometrie ist keine Hypothese, sondern eine Methode. Ihre Grundannahme der lokalen Gültigkeit der euklidischen Geometrie entspricht nach aller Erfahrung dem realen Raum.

    Nun zu QFT vs. ST. Die QFT hat eine ganz andere Qualität, von Anfang an. Sie ist natürlich abstrakt, aber wurde entwickelt in Korrespondenz zur Empirie. Durch die Übereinstimmung mit empirischen Befunden bekam sie Glaubwürdigkeit. Beispiele: Die Einführung der Eichsymmetrie und Gruppentheorie durch Weyl. Er fand auch die Gruppe U(1) für die Maxwellsche Elektrodynamik und die ist empirisch bestätigt. Als die QED vollendet war, wurde sie mit Erfolg am magnetischen Moment des Elektrons getestet.

    Die ST ist dagegen immer noch ein Konglomerat unbelegter Hypothesen. Hypothesen. gehören zu einer Theorie, aber es sollten so wenige (ich ergänze: auch so wenig weitreichende) wie möglich sein, umso größer sind die Erfolgschancen. Die ST ist ziemlich das Gegenteil davon. Ein Programm mit einem solchen Hypothesen-Aufbau dürfte es in der Physik noch nie gegeben haben, nicht annähernd. Wer das Ockhamsche Prinzip ernst nimmt und meint, dass es auch heute noch gilt, muss sich der ST-Theorie ggüb. eine gewisse Skepsis bewahren.

    Dabei hat die Konzeption zweifelsohne ihre eleganten Seiten, z.B. die vollständige Geometrisierung oder ungezwungene Lösungsansätze für das Hierarchieproblem (ich rede rede übrigens nicht von der angeblichen mathematischen Eleganz; das ist für mich kein physikalisches Argument; grob gesagt, Mathematik ist nicht mit Physik gleichzusetzen). Aber demgegenüber stehen die mangelnden spezifischen empirischen Belege. Besser gesagt, es gibt keine und so hängt die ganze ST-Geschichte quasi in der Luft und das tut sie, seit es sie gibt, seit über 30 Jahren. Vor allem am Hauptanspruch, nämlich das SM wiederzugeben scheitert sie und das nicht nur an Details, sondern die SUSY kommt nicht richtig heraus oder es treten weitere Wechselwirkungen mit unbegrenzter Reichweite auf. Das war jedenfalls der Stand vor ca. 10 Jahren, als Smolin seine Kritik schrieb. Ob man da inzwischen weitergekommen ist?

    Ferner sehe ich Elemente, deren Plausibilität für mich fragwürdig ist. Den String hatte ich schon erwähnt. Ein 1-dimensionaler Faden aus reiner Energie, der in sich schwingt, seltsam. Energie hat einen Träger, Schwingungen haben ein Medium. Ein Feld erfüllt beides, es kann Energie tragen und schwingen, aber reine Energie, die schwingt, was soll das? Man kann das so hinnehmen, auch gut, aber für mich bleiben Fragen. Auch bei der höherdimensionalen Einbettung frage ich mich, ob das physikalisch plausibel ist. Mathematisch ist das kein Problem, aber betrachten wir z.B. ein im 3-D-Raum eingebettetes 2-D-Objekt im Querschnitt: Ausdehnung = 0! Macht das überhaupt einen Sinn? Objekte sollte m.E. als erstreckt in alle Dimensionen betrachtet werden.

    Jetzt tut sich ja was, zumindest in Sachen Falsifizierung. Die Frage ist, ob sich die SUSY endlich im LHC zeigt oder nicht. Bis TeV sollte sie ja auftreten. Wenn nicht, was ist die Auswirkung auf die ST? Diese Frage habe ich schon öfters gestellt, aber nie eine Antwort bekommen. Wobei, eine Antwort kann ich mir denken: Die Symmetriebrechung ist stärker. Klar, die kann man höher treiben, aber mit der Glaubwürdigkeit der SUSY wird’s abwärts gehen. Auch weil sie ihre Vorteile dann verloren hat. Bereits jetzt ist die SUSY in der Kritik, da die sog. “natürliche SUSY” nicht aufgetreten ist (s. Artikel in Spektrum Spezial 1/15). Aber warten wir’s ab, es wird spannend …

  49. #49 kostuek
    30. Mai 2015

    Ein sehr interessanter Artikel, danke! Als interessierten Laien würde mich noch interessieren, wie es dann bei dem Einstein-Tensor aussieht? Also hier: ” Wichtig am Einstein-Tensor ist, dass hier auch Terme drinstecken, in denen die Raumzeitkrümmung sozusagen auf sich selbst wirkt – eine hinreichend stark gekrümmte Raumzeit wirkt auf sich selbst zurück.” Rein logisch müsste ja dann dieser auch “explodieren” wie ein sich selbst verstärkender Schwingkreis (falls ich das mit den Gravitonen richtig verstanden habe). Es gibt dort dann wohl noch Terme, die diesem Schicksal entgegenwirken? Und die dem Gravitonen fehlen? Sorry, wenn ich Unsinn frage 🙂

  50. #50 MartinB
    30. Mai 2015

    @Physik-Fan
    ” Ein experimenteller Befund ist keine Hypothese, sondern eine Tatsache. ”
    Das halte ich für eine sehr gewagte Aussage – letztlich ist ein experimenteller Befund ja so was wie ein zeigeraussdchlag auf einem Instrument – alles was darüber hinaus geht, bedarf einer Theorie.

    “Bspwse. galt die LG-Invarianz nach dem Michelson-Morley-Experiment als nachgewiesen. ”
    Das ist historisch falsch, soweit ich weiß – es gab z.B. Ideen, dass die Erde den Äther mitzerrt.

    “Vielleicht gibt’ s das auch schon irgendwo in ScienceBlogs. ”
    Zum Beispiel hier:
    https://scienceblogs.de/hier-wohnen-drachen/2010/12/10/physik-und-geist-dialog-uber-zwei-weltsysteme/

    “Interessant ist die Frage, ob eine notwendige Aussage auch als Hypothese betrachtet werden kann, da die Aussage ja aus Prämissen der Theorie ableitbar ist.”
    Da geht für mich jetzt die Begrifflichkeit stark von dem auseinander, was ich gewohnt bin – danach ist eine Hypothese eine mögliche Erklärung (also ein Erklärungsversuch), eine Theorie eine gut bestätigte Erklärung. Deswegen kann eine Theorie eigentlich keine Hypothesen enthalten (es sei denn, es geht darum, die Theorie zu erweitern). In dieser Begrifflichkeit ist die String-Theorie natürlich keine Theorie, sondern eine Hypothese, da unbelegt.

    ” Da ist doch zunächst mal die Frage, ob es überhaupt Belege für die Existenz höherer Dimensionen gibt.”
    Richtig – sehe ich auch als Problem. Aber das ist natürlich für sich noch kein Ausschlusskriterium, es wäre ja nicht das erste Mal, dass eine Theorie ein Phänomen vorhersagt, für das man vorher keine Anhaltspunkte hatte.

    “So gesehen sind Hypothesen mehr was für die Physik, Postulate mehr was für formale Systeme. ”
    Aber bei einer Theorie spricht man ja auch gern von Postulaten oder Axiomen (z.B. sind bei der SRT die Gleichberechtigung der Inertialsysteme und die Konstanz der Lichtgeschwindigkeit die beiden Postulate, aus denen man den rest herleiten kann.)

    “Die riemannsche Geometrie ist keine Hypothese, sondern eine Methode. ”
    Wieso das? Wenn die euklidische Theorie eine axiomatische Theorie ist, dann kann dieRiemannsche Geometrie (die ja lediglich ein Axiom modifiziert) nicht etwas anderes sein.

    ” Die QFT hat eine ganz andere Qualität”
    Da stimme ich voll zu.

    “Die ST ist dagegen immer noch ein Konglomerat unbelegter Hypothesen.”
    Sollte man nicht eher sagen: Sie ist eine Hypothese (ein Gedankengebäude, das es noch nicht zur Theorie geschafft hat), die auf bestimmten Postulaten beruht?

    Was sind denn nun grob die Hypothesen der Stringtheorie?

    “Ein Feld erfüllt beides, es kann Energie tragen und schwingen, aber reine Energie, die schwingt, was soll das?”
    Also ich kann mir ein (punktuell sogar divergentes) Feld wie in der QFT auch nicht besser vorstellen als einen 1D-String. Die anschauliche Plausibilität ist meiner Ansicht nach kein guter Ratgeber.

    “Objekte sollte m.E. als erstreckt in alle Dimensionen betrachtet werden.”
    Das gilt ja für Quantenfelder auch nicht – und die Strings werden ja auch quantisiert.

    Was die ST und Susy angeht – gab ja schon einen Kommentar, in dem wir lesen konnten, dass jemand schon mal vorsichtshalber ne ST-Variante ohne Susy entworfen hat. (Was übrigens auch wieder zeigt, dass die Interpretation von Experimenten nicht ohne eine Theorie, auf die sie sich bezieht, möglich ist.)

    @kostuek
    “Rein logisch müsste ja dann dieser auch “explodieren” wie ein sich selbst verstärkender Schwingkreis ”
    Nicht unbedingt. Für kleine Raumzeitkrümmungen werden die Effekte immer schwächer – man kann das vielleicht wie eine unendliche Summe betrachten, die aber gegen einen endlichen Wert geht (1+1/2+1/4+…=2). Erst wenn die Felder zu stark werden, explodiert das ganze, das nennt man dann “Singularität” – also genau das, was im Innern eines Schwarzen Lochs steckt.
    Und nein, die Frage ist kein Unsinn.

  51. #51 Physik-Fan
    11. Juni 2015

    @kostuek

    Ein sehr interessanter Artikel, danke! Als interessierten Laien würde mich noch interessieren, wie es dann bei dem Einstein-Tensor aussieht? Also hier: ” Wichtig am Einstein-Tensor ist, dass hier auch Terme drinstecken, in denen die Raumzeitkrümmung sozusagen auf sich selbst wirkt – eine hinreichend stark gekrümmte Raumzeit wirkt auf sich selbst zurück.” Rein logisch müsste ja dann dieser auch “explodieren” wie ein sich selbst verstärkender Schwingkreis (falls ich das mit den Gravitonen richtig verstanden habe). Es gibt dort dann wohl noch Terme, die diesem Schicksal entgegenwirken? Und die dem Gravitonen fehlen? Sorry, wenn ich Unsinn frage 🙂

    Das Grav.feld wirkt auf sich zurück, wegen der Energie-Massen-Äquivalenz. Das war für Einstein von Anfang an klar. Die Konsequenz war für ihn eine nichtlineare Feldgleichung, was sie auch ist. Das ist auch kein Problem, wenn die Gleichung sinnvolle Ergebnisse liefert und das tut sie bis auf Extremfälle, wo Singularitäten auftreten. Diese deuten daraufhin, dass die Theorie ihr Grenzen hat, weil Unendlichkeiten physikalisch sinnlos sind.

  52. #52 Physik-Fan
    11. Juni 2015

    @MartinB
    “Bspwse. galt die LG-Invarianz nach dem Michelson-Morley-Experiment als nachgewiesen. ”
    Das ist historisch falsch, soweit ich weiß – es gab z.B. Ideen, dass die Erde den Äther mitzerrt.

    Das stimmt, aber es war doch nur ein Rückzugsgefecht der Äthertheorie. Sie wurde immer vertrackter, hässlicher. Wohl die meisten Physiker glaubten nicht mehr so recht daran. Als Einstein mit seiner SRT kam, wurde sie schnell akzeptiert, obwohl empirische Belege für Zeitdilation etc. noch fehlten. Das bedeutet, dass man auch ihre Grundlage, das Relativitätsprinzip, akzeptierte.

    Da geht für mich jetzt die Begrifflichkeit stark von dem auseinander, was ich gewohnt bin – danach ist eine Hypothese eine mögliche Erklärung (also ein Erklärungsversuch), eine Theorie eine gut bestätigte Erklärung. Deswegen kann eine Theorie eigentlich keine Hypothesen enthalten (es sei denn, es geht darum, die Theorie zu erweitern). In dieser Begrifflichkeit ist die String-Theorie natürlich keine Theorie, sondern eine Hypothese, da unbelegt.

    Sehr gut, dass Du auf diesen Umstand hinweist! Der Begriff “Theorie” wird heute inflationär verwendet (man tut es ja selber auch, hat sich irgendwie daran gewöhnt). Dunkle Materie, Dunkle Energie, Inflation, SUSY, Axion, String-Theorie, LQG sind eigentl. nur Hypothesengebäude.

    “Die riemannsche Geometrie ist keine Hypothese, sondern eine Methode. ”
    Wieso das? Wenn die euklidische Theorie eine axiomatische Theorie ist, dann kann dieRiemannsche Geometrie (die ja lediglich ein Axiom modifiziert) nicht etwas anderes sein.

    Aus Sicht der ART ist es eine Methode, ein allg. Instrument zur Beschreibung von Räumen, die lokal einem euklidischen (bzw. pseudo-euklidischen) Raum gleichen. Auf dieser Entsprechung beruht die Anwendbarkeit dieser Mathematik.

    “Was sind denn nun grob die Hypothesen der Stringtheorie?

    Das hatte ich doch schon in #48 ausgeführt. Es ist eben nicht nur der String selbst, sondern es kommen eine ganze Reihe von Annahmen dazu, damit überhaupt was draus wird, wie die 10(11)-dimensionale Raumzeit und der Calabi-Yau-Raum. Aber, warum sollte das so sein? Antwort der ST: Weil es so funktioniert oder besser gesagt, funktionieren KÖNNTE* Begründungen für die String-Hypothese sind das nicht, sondern im besten Fall gut aufgezeigte notwendige Bedingungen. Das stärkt die String-Hypothese aber nicht, sondern schwächt sie. Für so weitreichende Annahmen sollte es eigenständige Argumente geben.

    *Dass wirklich die Realität von der Mikro- bin zur Makrophysik rauskommt, ist nicht nachgewiesen. Tatsächlich ist man von der Realität weit weg, es gibt ungelöste Widersprüche (zumindest Stand 2005 lt. Smolin). Hat man Argumente dafür, dass es in der String-Landschaft überhaupt eine korrekte Lösung gibt? Außer halt, dass die Landschaft so riesig ist, was ein schwaches Argument ist.

    ““Objekte sollte m.E. als erstreckt in alle Dimensionen betrachtet werden.”
    Das gilt ja für Quantenfelder auch nicht – und die Strings werden ja auch quantisiert.

    Das ist mir unklar. Ein Quantenfeld ist doch definiert auf der Raumzeit. In jedem Weltpunkt hat es einen Wert. Meinst Du vielleicht den Konfigurationsraum als Werteraum? Aber das ist doch was anderes.

  53. #53 Physik-Fan
    12. Juni 2015

    @MartinB
    “Ein Feld erfüllt beides, es kann Energie tragen und schwingen, aber reine Energie, die schwingt, was soll das?”
    Also ich kann mir ein (punktuell sogar divergentes) Feld wie in der QFT auch nicht besser vorstellen als einen 1D-String. Die anschauliche Plausibilität ist meiner Ansicht nach kein guter Ratgeber.

    Ein punktförmiges, strukturloses Teilchen ist schon was anderes als ein Gebilde, das in sich
    schwingt. Während das Erste weitgehend selbsterklärend ist, ist das Zweite komplex und es stellen sich sofort Fragen, z.B. was denn da schwingt. Mit der Antwort “reine Energie” kann ich wenig anfangen. Da bekommt Energie eine fast mystische Qualität.

    Außerdem kann der String nicht elementar sein, da er in sich beweglich ist. Er muss aus kleineren Einheiten bestehen. Vielleicht, jetzt spekuliere ich, muss man da tiefer schürfen, wie etwa die LQG. Deren Ansatz ist für mich attraktiv, weil sie bei der Raumzeit selbst ansetzt und eine tiefere, netzartige Struktur annimmt. In ihr ist die Raumzeit dynamisch (zumindest prinzipiell?), während sie in der ST starr vorgegeben ist. Es scheint keine ST-Gleichungen zu geben, die diffeomorphismus-invariant sind (kann es eigentl. nicht, denn wäre das QG-Problem bereits in Form der ST vollständig gelöst), was mich ist eine große Schwäche der ST ist, bis heute.

    Aber auch in der LQG scheint man irgendwie festzustecken, von signifikanten Fortschritten ist nichts zu hören. Das Gleiche gilt für die ST. Es scheint immer noch der Stand Anfang der 2000er Jahre zu sein, jedenfalls was die zentralen Probleme anbetrifft. Ist irgendeiner der Kritikpunkte von Smolin in 2005 geklärt worden? Er scheint sich aber inzwischen auch von der LQG verabschiedet zu haben. Auch experimentell gibt es bisher i.W. nur negative Resultate (z.B. sagt die LQG leichte Abweichungen von der LG voraus).

    Ein Grundproblem ist vielleicht, dass in der theoretischen Physik da zwei Lager werkeln. Die einen kommen von der Teilchenphysik her, die anderen von der Kosmologie. Die einen arbeiten auf Basis der QFT, die anderen auf Basis der ART. Zusammen kommen sie irgendwie nicht. Es gibt schon Leute, die beides verbinden wollen, wie Kiefer, aber das sind wohl nur wenige.

  54. #54 MartinB
    12. Juni 2015

    @Physikfan
    Sorry, war verreist, deswegen dauerte die ANtwort etwas…
    “aber es war doch nur ein Rückzugsgefecht der Äthertheorie.”
    Im Nachhinein wissen wir, dass das so war – ob das damals alle so gesehen haben, wage ich sehr zu bezweifeln.

    ” es kommen eine ganze Reihe von Annahmen dazu, damit überhaupt was draus wird, wie die 10(11)-dimensionale Raumzeit und der Calabi-Yau-Raum”
    Aber der 10-dimensionale Raum ist doch eine Konsequenz der Theorie, den steckt man ja nicht willkürlich rein. Insofern würde ich ihn nicht zu den Hypothesen zählen.

    “Während das Erste weitgehend selbsterklärend ist, ist das Zweite komplex und es stellen sich sofort Fragen, z.B. was denn da schwingt. ”
    Leider weiß ich nich tgenau, wie man die Strings am Ende quantisiert. Muss man dann überlagerungen aller denkbaren Stringkonfigurationen betrachten? Die Feynman-Hosen-Diagramme deuten das ja an. Dann sehe ich den großen Unterschied zwischen einer Überlagerung von Linien oder von Flächen nicht.
    Und was bei einem String schwingt – der String halt. Irgendwann ist man beim Erklären nun mal an der Grenze, weil man ja letztlich immer nur mathematische Beschreibungen bastelt. Wenn die Theorie gute Voerhersagen machen würde, dann könnte ich damit locker leben; unanschaulicher als ne qm-Überlagerung finde ich es nicht.

    “Mit der Antwort “reine Energie” kann ich wenig anfangen. ”
    Sagt das irgendwer (außer Spock). “Reine Energie” gibt es doch nur in der SF.

    “Er muss aus kleineren Einheiten bestehen. ”
    Naja, es könnte einfach ein konitnuierliches 1D-Objekt sein – die Idee der kontinuierlichen Materie ist zwar nicht mehr so aktuell, aber am Ende sind Quantenfelder auch kontinuierliche Objekte. (Immerhin gibt es einen Artikel mit dem Titel “There are no particles, there are only fields”)

    “Deren Ansatz ist für mich attraktiv, weil sie bei der Raumzeit selbst ansetzt und eine tiefere, netzartige Struktur annimmt.”
    Finde ich intuitiv auch überzeugender – aber am Ende entscheidet die Vorhersage.

    Ich glaube, da fehlt einfach noch eine ganz grundlegende Idee – so wie die von Penrose (die aber anascheinend ja nicht klappt), dass der QM-Kollaps was mit der Gravitation zu tun hat.

  55. #55 Physik-Fan
    16. Juni 2015

    @MartinB
    Aber der 10-dimensionale Raum ist doch eine Konsequenz der Theorie.

    Davon rede ich ja mit “notwendiger Bedingung”, nämlich wenn eine Aussage B aus einer Aussage A folgt. B ist dann prinzipiell allgemeiner gültig wie A, hat also schon einen eigenständigen Charakter. Außerdem, wenn B nicht gilt, dann gilt auch A nicht. Dadurch wird A abgeschwächt, denn sie hängt jetzt von B ab.

    Generell sollte man sich m.M.n. bei rein mathematischen Schlussfolgerungen eine gewisse Skepsis bewahren. Es hat sich schon öfters herausgestellt, dass es doch nicht so korrekt ist, weil nicht alle Voraussetzungen in der Form gelten oder weil es nicht streng und allgemeingültig abgeleitet ist. Zwei Beispiele:

    1. Der Vollständigkeitsbeweis der QM durch von Neumann. Jahrzehntelang hatte man angenommen, dass er korrekt ist, bis Bell darauf kam dass eine der Voraussetzungen unsauber ist.
    2. Die Unendlichkeiten der Supergravitation bei 3 Schleifen. Bereits Hawking hatte auf seiner Meinung nach unzulässige Vereinfachungen bei Berechnung der Feynman-Diagramme hingewiesen (ca. 1**20 Terme). Mit der Unitaritätsmethode (s. Spektrum 09/2012) konnte man die Endlichkeit auch für 4 Schleifen nachweisen. Man ist jetzt bei den Rechnungen für höhere Ordnungen.

    Die Unitaritätsmethode ist generell eine hochinteressante Sache. Es deutet sich z.B. eine Verwandtschaft zwischen Gravitation und Starker WW an. Das Graviton soll als zwei verbundene Kopien eines Gluons dargestellt werden können. Über Fortschritte bei der Unitaritätsmethode hört man leider nichts. Beim Nachweis des Higgs-Bosons hat sie jedenfalls eine wichtige Rolle gespielt.

    @MartinB
    “Mit der Antwort “reine Energie” kann ich wenig anfangen. ”
    Sagt das irgendwer (außer Spock). “Reine Energie” gibt es doch nur in der SF.

    Gelesen hab ich es schon (bei Green? bei Lüst? … müsste mal nachforschen). Dass das mehr SF ist, meine ich auch.

  56. #56 Niels
    16. Juni 2015

    @Physik-Fan

    Davon rede ich ja mit “notwendiger Bedingung”, nämlich wenn eine Aussage B aus einer Aussage A folgt. B ist dann prinzipiell allgemeiner gültig wie A, hat also schon einen eigenständigen Charakter.

    Hm? Wenn aus A folgt B gilt, ist B allgemein gültiger als A?
    Ganz sicher?

    Außerdem, wenn B nicht gilt, dann gilt auch A nicht. Dadurch wird A abgeschwächt, denn sie hängt jetzt von B ab.

    Weil aus A folgt B äquivalent ist zu aus Nicht-B folgt Nicht-A, hängt A auf einmal von B ab, wodurch A abgeschwächt wird?
    Wirklich?

    Generell sollte man sich m.M.n. bei rein mathematischen Schlussfolgerungen eine gewisse Skepsis bewahren.

    Welche Schlussfolgerungen aus Theorien gibt es denn, die nicht rein mathematisch sind?
    Meiner Meinung nach haben sich solche Schlussfolgerungen übrigens in der großen Mehrheit als richtig erwiesen, auch wenn sie zunächst sehr unsinnig erschienen.
    Spontan fallen mir da elektromagnetische Wellen, Neutrinos, der Urknall und die Hintergrundstrahlung, schwarze Löcher, Antiteilchen, Bose-Einstein-Kondensation, Supraleitung und zahlreiche Elementarteilchen wie zuletzt das Higgs-Teilchen.

    Ich weiß aber gar nicht, ob das hier überhaupt passt. Meiner Meinung nach ist der 10-dimensionale Raum nämlich eher ein Postulat als eine mathematischen Schlussfolgerung.
    Wie gesagt, ich habe aber praktisch keine Ahnung von der String-Theorie.

    Über Fortschritte bei der Unitaritätsmethode hört man leider nichts. Beim Nachweis des Higgs-Bosons hat sie jedenfalls eine wichtige Rolle gespielt.

    Echt? Inwiefern?

  57. #57 MartinB
    17. Juni 2015

    @Physik-Fan
    Deine Sprechweise kann ich nicht nachvollziehen – ist der Schwerpunktsatz eine notewndige Bedingung der Mechanik oder sind Gravitationswellen eine Bedingung der ART? (Dann hätte man 1920 genauso gegen die ART argumentieren können wie du jetzt gegen die ST.)
    Ich glaube, du vermischt zwei Konzepte: Die einer Theorie zu grunde liegenden Postulate/Axiome/konzepte auf der einen Seite.
    Auf der anderen Seite stehen dann die beobachtbaren Konsequenzen einer Theorie. Die sind die Bedingung, dass eine Theorie mit unserem Universum korrekt übereinstimmt, aber sie sind keine Bedingung der Theorie (des gedankengebäudes) selbst.
    Und es ist letztlich unfair, einerseits der ST vorzuhalten, sie würde keine Vorhersagen machen, und andererseits die klare Vorhersage der Dimension der Raumzeit konzeptionell zu einer Annahme der Theorie zu erklären. (Dass die Vorhersage nicht mit unseren Beobachtungen übereinstimmt, steht auf einem anderen Blatt.)

    Was die “reine Energie” angeht – in populärwissenscjhaftlichen Darstellungen steht oft begrifflich ziemlicher Blödsinn. Man kann evtl. von “reiner Energie” sprechen, wenn man die Paarvernichtung zweier Antiteilchen betrachtet – Im Ruhesystem und genau zum zeitpunkt der Vernichtung hat man einen Zustand, dessen einzige von Null verschiedene physikalische Größe die Energie ist (ladung, Impuls etc. heben sich alle auf). Aber auch das ist nur bedingt korrekt – denn in Wahrheit findet die Paarvernichtung ja nicht an einem genauen Raumzeitpunkt statt; man darf den vertex im feynman-Graphen ja nicht als echten Raumzeitpunkt auffassen, sondern muss über alle integrieren.

  58. #58 Physik-Fan
    17. Juni 2015

    @Niels
    Hm? Wenn aus A folgt B gilt, ist B allgemein gültiger als A?
    Ganz sicher?

    Betrachte es mengentheoretisch. Der Gültigkeitsbereich von A ist eine Teilmenge des Gültigkeitsbereichs von B. B kann darüberhinaus gültig sein. Im konkreten Fall kann die Raumzeit 10-dimens. sein, aber ohne Strings, während das Umgekehrte nicht gilt (wenn die Schlussfolgerung wirklich streng zutrifft).

    @Niels
    Weil aus A folgt B äquivalent ist zu aus Nicht-B folgt Nicht-A, hängt A auf einmal von B ab, wodurch A abgeschwächt wird?
    Wirklich?

    Nehmen wir an, dass die Schlussfolgerung streng zutrifft, d.h. Strings benötigen eine 10-dimens. Raumzeit. Wenn die aumzeit aber so nicht ist, sind die Strings falsifiziert. Je länger Indizien für die Existenz von Extradimensionen ausbleiben (bei immer feineren Messungen), umso geringer wird die Glaubwürdigkeit der String-Hypothese.

    @Niels
    Welche Schlussfolgerungen aus Theorien gibt es denn, die nicht rein mathematisch sind?

    Natürlich ist die Mathematik neben dem Experiment das entscheidende Hilfsmittel der Physik. Aber, dass ein mathematischer Schluss über einen physikalischen Umstand in die Irre führen kann, zeigen meine Beispiele, das Erste weil eine der Prämissen nicht stimmte, das zweite weil vereinfachende Annahmen im Rechenprozess sich als falsch herausstellten. Ich will noch erwähnen, dass es sich dabei um wichtige Fälle handelt und Konsens über die Korrektheit des Beweises herrschte.

    Man kann es auch einfacher sagen: Physikalische Aussagen kann man nicht beweisen, nur falsifizieren.

    @Niels
    Ich weiß aber gar nicht, ob das hier überhaupt passt. Meiner Meinung nach ist der 10-dimensionale Raum nämlich eher ein Postulat als eine mathematischen Schlussfolgerung. ?

    Man konnte schon lesen, dass die Theorie nur auf einer 10-D-Raumzeit funktioniert. Aber ist das so sicher? Lüst berichtet z.B. dass sie Versuche mit einer 4-D-Formulierung unternahmen (und noch machen?), wo die Extradimensionen als logische Dimensionen interpretiert werden.

    @Niels
    “Über Fortschritte bei der Unitaritätsmethode hört man leider nichts. Beim Nachweis des Higgs-Bosons hat sie jedenfalls eine wichtige Rolle gespielt.”
    Echt? Inwiefern? ?

    Das steht in dem von mir erwähnten Artikel in der Spektrum 09/2012 (“Mit einem Rechentrick zur umfassenden Theorie der Naturkräfte”). Es ging um die exakte Berechnung der Wahrscheinlichkeit von Hintergrundereignissen für eine der Nachweisreaktionen, wo ein Elektron, zwei Jets und ein Neutrino entstehen. In dem Artikel sind weitere Berechnungen genannt, z.B. betreffend einer möglichen DM-Lücke beim Z-Boson-Zerfall.

    Allg. gesprochen kann man mit der Unitaritätsmethode Feynman-Diagramme stark vereinfachen, sodass sie berechenbar werden. Bei einer WW mit einer größeren Kopplungskonstante (z.B. Starke WW) ist die große Anzahl von Termen ja genau das Problem. Etwa die ca. 10^20 Terme bei der Supergravitation mit 3 Schleifen lassen sich zu ein paar Dutzend Rechenschritte reduzieren. Für mich sieht es danach aus, dass die Unitaritätsmethode einen enormen Fortschritt in der Teilchenphysik darstellt . Das aber nicht nur in praktischer Hinsicht, es deutet sich ein ungeahnter neuer Zugang zur QG an, wie schon erwähnt. Das wäre sensationell.

  59. #59 Physik-Fan
    22. Juni 2015

    @MartinB
    Deine Sprechweise kann ich nicht nachvollziehen – ist der Schwerpunktsatz eine notewndige Bedingung der Mechanik oder sind Gravitationswellen eine Bedingung der ART? (Dann hätte man 1920 genauso gegen die ART argumentieren können wie du jetzt gegen die ST.)
    Ich glaube, du vermischt zwei Konzepte: Die einer Theorie zu grunde liegenden Postulate/Axiome/konzepte auf der einen Seite.

    Und es ist letztlich unfair, einerseits der ST vorzuhalten, sie würde keine Vorhersagen machen, und andererseits die klare Vorhersage der Dimension der Raumzeit konzeptionell zu einer Annahme der Theorie zu erklären. (Dass die Vorhersage nicht mit unseren Beobachtungen übereinstimmt, steht auf einem anderen Blatt.)

    Wenn die Grav.wellen wirklich streng folgen und nachweisbar wären, dann wäre durch ein negatives Resultat die Theorie in Frage gestellt. Es sind ja genauere Experimente geplant, indirekte Hinweise gibt es schon. Für mich sind Grav.wellen auch nicht so überraschend. Wenn die Raumzeit dynamisch ist und sich wie ein biegbares Medium verhält, warum sollte es keine wellenartigen Prozesse darin geben? Zumal die Raumzeit extrem steif ist und sich Wellen darin weiträumig ausbreiten können. Grav.wellen beziehen sich auch auf den Anwendungsbereich der Theorie und sind nur eine weitere Aussage, über das, was die Theorie beschreibt, nämlich die Raumzeit.

    Hypothesen gehören zu einer Theorie, aber es ist die Frage, wie viele es sind und wie weitreichend sie sind. Beides sollte so gering wie möglich sein. Hierin liegt für mich das Hauptproblem bei der ST. Sie geht in dieser Hinsicht so weit, wie kein anderes Programm in der Physikgeschichte. Die 10-dimen. Raumzeit dürfte die weitreichendste physikalische Aussage sein, die jemals gemacht wurde. Davon auszugehen, ohne dass es einen empirischen Hinweis darauf gibt, ist gewagt. Dabei ist die 10-dimen. Raumzeit kein Gegenstand der Theorie, da die ST keine Theorie der Raumzeit ist. Sie benötigt nur diesen Rahmen, damit sie funktioniert (= anomaliefrei ist). Das kommt also als Voraussetzung für die Theorie dazu. Was ist, wenn die Raumzeit so nicht ist? Für die Raumzeit wäre das kein Problem, aber für die ST.

    Beim Calabi-Yau-Raum verhält es sich anderes. Er ist meines Wissens nicht abgeleitet, sondern nur hinreichend dafür, dass das SM herauskommt. Genauer gesagt herauskommen KÖNNTE, denn dass es so ist, ist nicht nachgewiesen (es “klemmt” immer noch ziemlich). Calabi-Yau ist eine Hypothese. Spricht irgendwas dafür, dass es genau Calabi-Yau ist? Warum sollte es nicht andere Kompaktifzierungen geben, die dem SM genügen? Für mich es merkwürdig, dass man sich nur auf diese Kompaktifzierung beschränkt.

    Man kann evtl. von “reiner Energie” sprechen, wenn man die Paarvernichtung zweier Antiteilchen betrachtet – Im Ruhesystem und genau zum zeitpunkt der Vernichtung hat man einen Zustand, dessen einzige von Null verschiedene physikalische Größe die Energie ist (ladung, Impuls etc. heben sich alle auf). Aber auch das ist nur bedingt korrekt – denn in Wahrheit findet die Paarvernichtung ja nicht an einem genauen Raumzeitpunkt statt; man darf den vertex im feynman-Graphen ja nicht als echten Raumzeitpunkt auffassen, sondern muss über alle integrieren.

    Feynman-Diagramme sind doch reine Teilchenprozesse. Bei der Annihilation verschwinden Teilchen und Antiteilchen und Photonen entstehen. Sind nicht zu jeder Zeit Teilchen da?

  60. #60 MartinB
    23. Juni 2015

    @Physikfan
    “Das kommt also als Voraussetzung für die Theorie dazu. Was ist, wenn die Raumzeit so nicht ist? ”
    Dass die Folgerung eine sehr starke ist, darin sind wir uns ja einig. Ich würde sie aber nach wie vor nicht als “Voraussetzung der Stringtheorie” beschreiben, sondern allenfalls als “notwendige Bedingung für die Gültigkeit”. Und das ist in meinen Augen absolut nicht dasselbe.
    Einer Theorie quasi vorzuwerfen, dass sie weitreichende und verblüffende Vorhersagen macht, würde ja im Endeffekt bedeuten, dass wir Theorien mit möglichst wenig Vorhersagekraft vorziehen müssten (bzw. solche, die nur vorhersagen, was wir schon wissen).

    Anders gesagt: Ich habe kein Problem mit dem, was du physikalisch sagst – aber wissenschaftsphilosophisch/-theoretisch halte ich deine Sprechweise für nicht vertretbar.

    Die Frage mit den Feynman-Diagrammen verstehe ich nicht – und nein, es sind nicht zu jedem Zeitpunkt Teilchen da, schließlich gibt es Teilchen gar nicht (oder genauer gesagt, es gibt sie nur als asymptotischen Grenzfall der nicht wechselwirkenden Theorien, wenn man die QFT wirklich ernst nimmt), es gibt nur Felder.

  61. #61 Physik-Fan
    25. Juni 2015

    @MartinB
    “Das kommt also als Voraussetzung für die Theorie dazu. Was ist, wenn die Raumzeit so nicht ist? ”
    Dass die Folgerung eine sehr starke ist, darin sind wir uns ja einig. Ich würde sie aber nach wie vor nicht als “Voraussetzung der Stringtheorie” beschreiben, sondern allenfalls als “notwendige Bedingung für die Gültigkeit”. Und das ist in meinen Augen absolut nicht dasselbe.
    Einer Theorie quasi vorzuwerfen, dass sie weitreichende und verblüffende Vorhersagen macht, würde ja im Endeffekt bedeuten, dass wir Theorien mit möglichst wenig Vorhersagekraft vorziehen müssten (bzw. solche, die nur vorhersagen, was wir schon wissen).

    Anders gesagt: Ich habe kein Problem mit dem, was du physikalisch sagst – aber wissenschaftsphilosophisch/-theoretisch halte ich deine Sprechweise für nicht vertretbar.

    Unsere unterschiedl. Betrachtungsweisen kommen jetzt gut heraus. Du betrachtet das als Folgerung, während ich es von der anderen Seite her, als notwendige Bedingung sehe. Die 10-dimens. Raumzeit wird der Theorie hinzugefügt, damit sie funktioniert. Die Struktur der Raumzeit geht aus der Theorie nicht hervor, sondern sie ist als äußerer Rahmen vorgegeben. Mir widerstrebt es schon, das als Folgerung zu betrachten, weil es mehr von der Art “es geht nur so und nicht anders” ist.

    Bei den Grav.wellen ist es anders gelagert. Das ist eine gefolgerte Eigenschaft der Raumzeit, also des Gegenstands der Theorie. Es wird der Theorie nicht was hinzugefügt, sondern ihre Aussagen weiter ausgebaut.

    @MartinB
    Die Frage mit den Feynman-Diagrammen verstehe ich nicht – und nein, es sind nicht zu jedem Zeitpunkt Teilchen da, schließlich gibt es Teilchen gar nicht (oder genauer gesagt, es gibt sie nur als asymptotischen Grenzfall der nicht wechselwirkenden Theorien, wenn man die QFT wirklich ernst nimmt), es gibt nur Felder.

    Feynman-Diagramme handeln doch von Teilchen, ihren Pfaden und Reaktionen sowie den zugehörigen Wahrscheinlichkeitsamplituden. Das ist es, was ich angesprochen habe. Ein Diagramm ist natürlich nicht als realistischer Prozess aufzufassen, sondern erst alle möglichen Diagramme zusammen ergeben den vollständigen Vorgang. Dadurch wird das Teilchenbild obsolet. In Experimenten treten aber Teilchen auf und nur Teilchen. In der QFT ist das Quantenfeld die grundlegende Entität, schon klar. Es müssen in ihr aber auch Teilchen repräsentiert sein. Ich gebe zu, wie das im Einzelnen geht, ist mir nicht ganz klar. Beispielweise ist es die Frage, ob es für Elektronen ein universumweit ausgebreitetes Feld gibt oder für jedes Elektronen ein Feld oder eine Mischform, wo allen Elektronen eines Systems (z.B. eine Elektronenhülle) ein Quantenfeld zugeordnet ist. Vielleicht hast Du über diese Zusammenhänge auch schon was geschrieben.

  62. #62 MartinB
    25. Juni 2015

    @Physik-Fan
    ” Du betrachtet das als Folgerung, während ich es von der anderen Seite her, als notwendige Bedingung sehe.”
    Wie gesagt, es ist eine Folgerung aus dem Theoriegebäude und damit eine notwendige Bedingung dafür, dass die Theorie auf unser Universum zutrifft.

    ” Die Struktur der Raumzeit geht aus der Theorie nicht hervor, sondern sie ist als äußerer Rahmen vorgegeben. ”
    Aber die Theorie sagt doch zumindest die Dimension der RZ hervor – hätte sich hier 3+1 ergeben statt10+1 (oder 11+1), hätte man das doch sicher (und zu Recht) als Vorhersage betrachtet, die sich erfüllt hat. So ist es eben eine Vorhersage, die momentan mit der Realität nicht übereinstimmt, sondern nur mit Zusatzannahmen in Einklang gebracht werden kann.
    Ich würde also eher sagen, dass es nicht die 10 Dimensionen sind, die die Zusatzannahme bilden, sondern das Aufrollen oder sonstige Wegdiskutieren der zusätzlichen Dimensionen.

    Was die teilchen im QFT-Bild angeht – dazu steht ja viel in den neuen Artikeln zu Photon und Graviton, und auch ein bisschen in der QFT-Serie.

    Ich bin aber gerade wieder am massiv Nachdenken über die QFT – ich hoffe, daraus wird demnächst auch mal wieder ein Artikel zum Thema, der vielleicht auch die Frage aufgreift.

  63. #63 Funktionalistiker
    Sachsen
    7. Juli 2015

    Sie schreiben:
    “Eine Quantentheorie der Gravitation (kurz Quantengravitation oder QG), wird oft als der “Heilige Gral ” der Physik bezeichnet – bisher ist es aber noch niemandem gelungen, ihn zu finden.”
    Das stmmt so nicht.
    Unter primärer Beachtung und exakter Würdigung des materiellen Befundes findet man die funktionalen Grundlagen der Quantenphysik der Gravitation in der Mathematik der Relativitätstheorie.

    Sicher braucht man etwas Intuition dazu. Aber am Ende ist es doch relativ einfach – aber eben ganz anders, als man es jetzt denkt..

  64. #64 MartinB
    7. Juli 2015

    @Funktionalistiker
    Die (allgemeine) Relativitätstheorie sagt doch gerade nichts über Quanteneffekte der Gravitation aus?
    Und was soll die “exakte Würdigung des materiellen Befundes” sein?

  65. #65 Funktionalistiker
    Sachsen
    7. Juli 2015

    Die Allgemeine Relativitätstheorioe liefert nur einen kleinen Teil der Erkenntnisse für die Quantenphysik der Gravitation. Der wesentliche Teil der Quanteneffekte der Gravitation versteckt sich in der Mathematik der Speziellen Relativitätstheorie, hat aber seinen Niederschlag in der Quantenphysik selbst noch nicht gefunden.
    Der Weg der Erkenntnis beginnt an seinem Anfang und nicht an seinem Ende. Und man muss an seinem Anfang beginnen, sonst kommt man nicht ans Ziel, immer aber durch die exakte Würdigung dessen, was man gemessen und unter den unterschiedlichsten Perspektiven funktional betrachtet hat.

  66. #66 MartinB
    7. Juli 2015

    @Funktionalistiker
    O.k., wenn du nicht konkret antworten willst, dann ebennicht.

  67. #67 Funktionalistiker
    Sachsen
    8. Juli 2015

    Bitte nicht so ungedultig sein und an die vergangenen 100 Jahre denken.
    Ich habe dir mal aufgeschrieben wie ich die Verbindung zwischen RT und Quantenphysik hergestellt habe.
    Wir erhalten über die Gleichung E = m c² die Verbindung zwischen Relativitätstheorie und Quantenphysik
    Quantenphysikalisch darf das Atom betrachtet werden als stehende Welle mit der Masse m, der Energie E und der Frequenz fa.
    Nach der SRT geht in einem bewegten System ebenso wie nach der ART in einem Gravitationsfeld die Zeit langsamer.
    Im Messprozess ist mit der Atomuhr alternativ bestimmbar:
    1. die Zeit an den Orten der Messung / die Raumzeit t
    2. mit Außenbetrachtung die Frequenzen f
    3. durch einfache Zählung die Schwingungen
    Aus dem Gleichungssystem der SRT ist unter Nutzung der mit Außenbetrachtung gemessenen Frequenzen aber auch durch Zählung der Schwingungen die Geschwindigkeit eines Atoms (bewegtes System nach der SRT)jetzt als raumgreifende Welle mit der Eigengeschwindigkeit v nach dem Gleichungssystem y = f (x) wie folgt zu bestimmen:
    – für die Eigengeschwindigkeit des Atoms als Funktion der Frequenz der Welle des Atoms
    v = c √(1-f²:fa²)
    oder für die Frequenz der Welle des Atoms als Funktion der Eigengeschwindigkeit
    f = fa √(1-v²:c²)
    fa = Frequenz der stehenden Welle des Atoms
    f = Frequenz der Welle des Atoms im raumgreifenden Zustand
    c = Lichtgeschwindigkeit
    v = Eigengeschwindigkeit der Welle des Atoms im raumgreifenden Zustand

    Die nach der ART langsamer gehende Zeit (Raumzeit ), alternativ die verringerte Frequenz in der Außenbetrachtung bzw. die verringerte Anzahl der Schwingungen proportional zu den beteiligten Massen und zu deren Abstand versetzt das Atom als stehende Welle in einen raumgreifenden Zustand.
    Aus der ART sind übertragbar:

    – die Krümmung der Raumzeit als Funktion der (Atom-) Frequenz in Abhängigkeit von Masse und Abstand
    – die Krümmung des Raumes als kräftefreie Bahn sich bewegender Massen.

  68. #68 MartinB
    8. Juli 2015

    @Funktionalistiker
    “Quantenphysikalisch darf das Atom betrachtet werden als stehende Welle mit der Masse m, der Energie E und der Frequenz fa.”
    Nein. Man kann versuchen, Elekteronen so zu betrachten (Bohrsches Modell), aber das ist nicht allgemeingültig.
    Der Rest des Textes ist dann vollkommen abwegig.

  69. #69 Funktionalistiker
    Sachsen
    10. Juli 2015

    Nicht allgemeingültig and vollkommen abwegig sind keine Begründungen. In der Funktionalistik beginnt man immer mit dem materiellen Befund, nicht mit unbegründeten Aussagen.

    Also lass es dir gut gehen.

  70. #70 MartinB
    10. Juli 2015

    Du brauchst doch in deiner “Theorie” nur mal zu überlegen, warum sich die Elektronen in einem Wasserstoff- und einem Deuterium-Atom praktisch nicht unterscheiden, obwohl die Masse des zweiten Atoms doppelt so groß ist. Dann siehst du sofort, dass da etwas nicht stimmt.
    (Mal davon abgesehen, dass Worte wie “raumgreifender Zustand” zwar nett klingen, aber keine physikalische Aussage haben).

  71. #71 Funktionalistiker
    Sachsen
    11. Juli 2015

    Ich fange bei einer “Theorie” nie in der Mitte eines Weges zur Erkenntnis an.
    Also beginnen wir mal mit einem Anfang.
    Du hast eine Kugel aus Eisen – 10 kg- in der Hand. Die Kugel ist schwer.
    Du willst wissen, warum die Kugel schwer ist.
    Um zu der Erkenntnis zu kommen, finde an erster Stelle alle Antworten, die das, was du in der Hand hast beschreiben, und zwar in unterschiedlichste Art und Weise.

    Mach mal, wenn du den Weg gehen willst!
    Was ist das, was du in der Hand hälst? Antworte möglichst strukturiert. Vermeide sinnlose Antworten.

  72. #72 MartinB
    11. Juli 2015

    @Funktionalistiker
    Oben hast du eine Idee formuliert, die, wenn ich es richtig verstehe, darauf beruht, zu sagen, dass die Energie in einem Atom etwas mit dr Zeitdilatation im Schwerefeld zu tun hat.
    Ich habe dir mit dem Isotopenargument gezeigt, warum das nicht haltbar ist.
    Und jetzt kommt als Antwort ein “, finde an erster Stelle alle Antworten, die das, was du in der Hand hast beschreiben, und zwar in unterschiedlichste Art und Weise.”

    Sorry, aber das hat wirklich nichts mit Physik zu tun.

  73. #73 Funktionalistiker
    Sachsen
    11. Juli 2015

    Das hat noch nichts mit Physik zu tun, ist aber der Ausgangspunkt für den Weg zur Erkenntnis, den ich zwingend beschreiben will oder muss – der umfangreiche materielle Befund!
    Das Isotopenargument ist für mich nicht zielführend, weil Gravitation vom Atomkern also von den Neutronen und Protonen ausgeht.
    Elektronen sind verantwortlich für den Elektromagnetismus in der Form, die wir Anziehung und Abstoßung nennen -spielen sicher eine Nebenrolle bei der Gravitation.

    (Es gibt aber auch Physiker oder vielleicht Mathematiker, für die ist das ganze Universum nur ein Hohlogramm.)

    Also:
    Die Antwort müsste strukturiert lauten:
    1. Die Eisenkugel ist Teil:
    des Universums, Teil des Galaxiensuperhaufens, Teil des Galaxienhaufens, Teil der Örtlichen Gruppe, Teil der Milchstraße, Teil des Sonnensystems, Teil des Erde-Mond Systems, Teil der Erde.
    2. Die Kugel ist::
    3. Die Kugel bestehend aus:
    bis hin zu den Wirks von Dürr!
    Kannst du 2. und 3. beantworten.

  74. #74 MartinB
    11. Juli 2015

    @Funktionalistiker
    Ich habe keinen Schimmer mehr, was du eigentliuch willst. Wenn die Elektronen eine Nebenrolle spielen, was soll dann die Rechnung mit der Frequenz, die irgendwas mit der Zeitdilatation zu tun haben soll?

    “2. Die Kugel ist::”
    Die Frage (die keine ist) ist zu unspezifisch, als dass man sie beantworten könnte. Die Kugel kann eine Kanonenkugel und damit ein Mordinstrument sein, eine heilige Relique des ordens der Kugelanten, ein Kunstwerk, eine Version des Normkilogramms, ein Rohstoff, ein physikalisches Objekt, usw.

    “3. Die Kugel bestehend aus:”
    Je nachdem, auf welcher Skala du guckst. Körner, Versetzungen, Leerstellen, Atome, Elektronen Protonen, Neutronen, Quarks, Quantenfelder…

    Die Kunst bei der Physik besteht darin, spezifische, experimentell zumindest prinzipiell beantwortbare Fragen zu stellen.

    Die “Wirks” von Dürr haben mit Physik übrigens ebenfalls nicht viel zu tun, fürchte ich.

  75. #75 Funktionalistiker
    Sachsen
    12. Juli 2015

    Auf die experimentelle Feststellung , dass Teilchen Welleneigenschaften besitzen gab es wohl schon einen Nobelpreis.
    2. Die Kusgel ist-
    – ein Objekt, makroskopischer Körper
    – Materie – Element Eisen
    – Masse 10 kg, Energie (rund) 9 x 10 hoch 17 kg m² / s²
    – Summe der Atome
    Die Kugel als Kanonenkugel brauchen wir vielleicht später.
    3. Die Kugel besteht aus
    Atomen, die unterstrukturiert sind. Die Unterstrukturen sind das Proton, das Neutron und das Elektron.
    Die Bildung von Atomen erfolgt aus Energie, die Auflösung von Atomen liefert wieder Energie.
    Was sollte also drin sein außer Energie? und die ist darin gewaltig in Bewegung! Aus die Feststellung von Welleneigenschaften hatte ich bereits verwiesen.

    Ich habe für mich daraus auch das Übergeordnete festgestellt:
    Das Universum existiert, es besteht aus Objekten und diese befinden sich in Bewegung.
    Ich setze fort mit der Frage an dich:
    Was ist über Gravitation (Trägheit und Schwere) bekannt? Es geht wiederum um den materiellen Befund!
    Ich fange mit 2 (zum Teil) falschen Grundsatzaussagen an:
    1. Massen ziehen sich gegenseitig an.
    2. Unterschiedliche Massen fallen gleich schnell.
    Was ist dir noch an materiellen Befunden bekannt?
    Fahre bitte fort!

    Wir brauchen das ehe wir in die Physik / Mathematik einsteigen.

  76. #76 MartinB
    12. Juli 2015

    @Funktionalistiker
    Irgendwas hast du falsch verstanden – das hier ist die Kommentarspalte eines Blogs, keine Physik-Nachhilfe für dich privat.
    Zum Thema Gravitation habe ich hier diverse Texte geschrieben, spezifische Frage beantworte ich gerne, aber mehr auch nicht.

  77. #77 Funktionalistiker
    Sachsen
    12. Juli 2015

    ok.
    dann ENDE!

  78. #78 erik d ..
    12. Juli 2015

    #61 – #62
    Alles (das Universum als Entität) besteht aus Quantenfeldern (Physik) und deren mathematische Beschreibung (Information).
    Streben nach energiearmen Zuständen: … ein wichtiges grundlegendes Prinzip der Physik: Physikalische Systeme streben einen möglichst energiearmen Zustand an.
    -> es existiert ein Quantenfeld (QF) als Entität als energiearmer Zustand . . . ..
    -> dieses Entität-QF birgt in sich alle Informationen, welche das Universum als Entität ausmachen.
    -> das Universum als Entität stellt einen energiereicheren Zustand dar.
    –> d.h. ein Entität-QF stellt keine Energie dar, sondern Information als Entität
    –> Information als Entität im QF besitzt keine Form
    -> erst ein Zwang, der formlosen Information im Entität-QF eine Form mit diskretem Inhalt zu geben, lässt Energie entstehen.
    -> Energie in physikalischen Systemen streben einen möglichst energiearmen Zustand an.
    -> Der diskrete Inhalt ( ein mathematischer Formalismus, der den angeregten Zustand des QF darstellt) verhindert, zurück in den energielosen Zustand des Entität-QF zu gelangen.
    -> exp(i*π)+1=0 ist der math. Formalismus, welcher den Zwang aufrecht erhält, der auf das Entität-QF wirkt . . . ..
    –< es ist die Information, die jedes QF in sich trägt, um wieder zum Entität-QF zurück zu kehren
    –> Zwang und Entität-QF bilden einen Überlagerungszustand . . . ..welcher zur Interpretation der Quantengravitation führt . . . ..

  79. #79 Rainer
    Gießen
    14. Oktober 2015

    @MarinB
    Müssten sich nicht im zweiten Feynman-Diagramm die beiden Massen nach Austausch eines Gravitons einander annähern?

  80. #80 Rainer
    Gießen
    14. Oktober 2015

    @Funktionalistiker; @Physik-Fan
    Vielleicht tue ich Euch Unrecht, aber Ihr vermittelt MIR den Eindruck als wolltet Ihr Euch mit angelesenem Halbwissen profilieren.
    @erik d ..
    ?
    Vielleicht können wir uns darauf einigen, die Entitäten im Sinne von Heidegger als “Heineingeworfen ins Sein” zu interpretieren. Es könnte sich dann aber herausstellen, dass die Eulersche Identität exp(i*π)+1=0 eine epistemologische Grenze darstellt und wir an dieser Stelle passen müssen.

  81. #81 MartinB
    15. Oktober 2015

    @Rainer
    Ob sich zwei Objekte anziehen, wenn sie ein “virtuelles” Teilchen in einem Feynman-Diagramm austauschen, hängt vom Spin des Austauschteilchens ab. Bei geradzahligem Spin verringert der teilchenaustausch die energie, wenn sich die teilchen annähern, bei ungeradzahligem Spin erhöht er sie. Deshalb stoßen sich gleiche Ladungen ab, aber Massen ziehen sich an.
    Ausführlich habe ich das in meiner Quantenfeldtheorie-Serie erklärt (siehe recht sbei Artikelserien).
    Die Feynman-Diagramme darf man in der Hinsicht nicht wörtlich nehmen, weil man ja in einem Feynmandiagramm über alle Möglichkeiten addieren muss, das Bild ist nur symbolisch und soll es einem leicht machen, zu sehen, welche Integrale man zu lösen hat. (man würde es genauso zeichnen, wenn man zwei entgegengesetzte Ladungen hätte).
    Siehe
    https://scienceblogs.de/hier-wohnen-drachen/2010/10/09/wie-funktionieren-feymandiagramme/
    https://scienceblogs.de/hier-wohnen-drachen/2010/10/10/wie-funktionieren-feymandiagramme-teil2-wir-jagen-elementarteilchen/
    https://scienceblogs.de/hier-wohnen-drachen/2013/08/25/verfuhrerisch-einfach-feynmandiagramme/

  82. […] die Energien der Gravitonen winzig klein. (Mehr über Gravitonen und Quantengravitation findet ihr hier und hier.) Solche winzigen Quanten zu messen, wäre extrem […]

  83. #83 Laie
    Vietnam
    3. September 2016

    Ich würde jetzt nicht gerade behaupten das ganze auch nur annähernd zu verstehen…..
    Aber dennoch eine ungewöhnliche Frage, auch wenn die Begriffe etwas abgedroschen klingen mögen:

    Was wäre wenn Gravitonen sich aus welchem Grund auch immer gar nicht bewegen, sondern quasi orts(zeit)fest mit der Raumzeit verbunden sind und “warten” bis die anderen Teilchen “vorbeifliegen” um mit ihnen zu wechselwirken…..
    Würde das das Problem mit den unendlichkeiten lösen?
    Vermutlich funktioniert dann aber die Ausbreitung einer Gravitationswelle nicht mehr, es sei denn es gäbe noch ein Teilchen das die Kraft vermittelt….

  84. #84 MartinB
    3. September 2016

    @Laie
    Zum einen sind Gravitonen masselose Teilchen; die können also niemals ortsfest sein.

    Zum anderen ist “ortsfest” schon in der Newtonschen Mechanik nicht wirklich definiert – zwei mit gleichförmiger Geschwndigkeit relativ zueinander bewegte bezugssysteme sind gleichberechtigt; was für den einen ortsfest ist, ist es für den anderen nicht.

  85. #85 LUCA
    12. Dezember 2018

    “die Austauschteilchen der starken Kernkraft sind zwar masselos”

    Sind die Pionen nicht die Austauschteilchen der starken Kernkraft, mit einer Masse von 134,97 – 139,57 MeV?

  86. #86 rolak
    12. Dezember 2018

    Pionen?

    Nee LUCA, da verwechselste was. Das wären mehr so die Gluonen, bei Dpedia immerhin als “(theoretisch) 0 kg” einsortiert.

  87. #87 LUCA
    13. Dezember 2018

    Nein, ich meine starke Kernkraft bei Proton-Proton WW, das sogenannte YUKAWA Potential.

  88. #88 LUCA
    13. Dezember 2018

    Nein, ich meine die starke Kernkraft bei Proton-Proton WW, das sogenannte YAKAWA Potential.

  89. #89 LUCA
    13. Dezember 2018

    Nein, ich meine die starke Kernkraft bei Proton-Proton WW, das sogenannte YUKAWA Potential.

  90. #90 rolak
    13. Dezember 2018

    Yukawa

    Ein spontaner ModellWechsel ist zwar für die Bekleidungsmode ziemlich typisch, LUCA, jedoch in der Diskussion einer Theorie unangemessen, insbesondere wenn kontextbefreite Schnippel auf den Tisch gestreut werden.

  91. #91 MartinB
    13. Dezember 2018

    @LUCA
    Ja, man kann die Kraft zwischen Protonen/Neutronen über Pionenaustausch ganz gut modellieren. Der Begriff “starke Kernkraft” wird aber zumindest heutzutage für die Kraft durch die Gluonen, die die Protonen/NEutronen zusammenhalten, verwendet.

  92. #92 LUCA
    13. Dezember 2018

    @rolak:
    Ich weiß es ehrlich gesagt nicht, was Du hast.
    “insbesondere wenn kontextbefreite Schnippel auf den Tisch gestreut werden.”

    1. Kontextbefreite Schnippel habe ich nicht gefragt. Es geht immer noch um die starke Kernkraft.
    2. Dich habe ich ja gar nicht gefragt.

  93. #93 LUCA
    13. Dezember 2018

    @MartinB:
    Danke für die Klarstellung. Ich sah in einigen Büchern auch in neueren, dass die Pionen als Austauschteilchen der starken Kernkraft angegeben werden, deshalb die Frage. VG

  94. #94 MartinB
    13. Dezember 2018

    @rolak
    Wie du siehst kommt dein Ton nicht immer optimal an…

    @LUCA
    Kein Problem, das ist in der Physik ja üblich, Begriffe unscharf oder doppelt zu besetzen…

  95. #95 rolak
    13. Dezember 2018

    kommt dein Ton nicht immer optimal an

    Selbstverständlich nicht, MartinB, die allerwenigsten Menschen sind erfreut, wenn ihnen ihr verfehltes Verhalten auch nur halbwegs angemessen vorgespiegelt wird.

    Oder ist Deiner Meinung nach bereits #86 als negativ einzusortieren?

  96. #96 LUCA
    13. Dezember 2018

    @rolak
    Du lebst wohl in einer paranoiden Welt.
    “wenn ihnen ihr verfehltes Verhalten”
    Welches verfehltes Verhalten genau? Du Solltest das ganze mit etwas mehr Demut angehen. Deine Schreibkultur lässt viel zu wünschen übrig.

    @MartinB: Vielen Dank für Ihre Antwort. Ich hatte die Pionen als Quanten des starken Wechselwirkungsfeldes kennengelernt. Noch mal danke für die schönen Blogs.

  97. #97 MartinB
    13. Dezember 2018

    @rolak
    #86 ist mein Humor, aber den Vorwurf der kontextbefreiten Schnipsel in 90 kann ich auch nicht mehr nachvollziehen – die Frage “Was ist denn nun das Austauschteilchen der starken Kernkraft” braucht ja nicht wirklich Kontext… Mich persönlich stört der Ton trotzdem nicht, (da haue ich im persönlichen Umgang ganz andere Dinger raus…), aber ich sehe schon, dass man das als Angriff wahrnehmen kann

  98. #98 Panthauer
    18. April 2019

    Ich habe mir das jetzt nochmal durchgelesen:
    Wieso können zwei „virtuelle Gravitonen“ – die aus einem Graviton entstanden sind – noch ein Graviton austauschen und dieses „neue Graviton“ kann wieder eins ausstoßen u. s. w., bis unendlich viele herumschwirren, zu einem „schwarzen Loch“ werden und peng, aus der Traum? Ein Graviton entsteht doch bestimmt aus der Kollision zweier Teilchen (vielleicht einem Elektron und einem Positron?), die ganz bestimmt keine „unendliche Energie“ hatten………..wie soll jetzt das Graviton, das aus den beiden entsteht unendlich viel Energie haben, um all das „entstehen“ zu lassen…….?
    Die beiden Teilchen, die beim ersten Prozess kollidiert sind, waren doch REAL? Oder habe ich da was falsch verstanden?

  99. #99 MartinB
    18. April 2019

    @Panthauer
    Während der virtuellen Prozesse können die Energien beliebig hoch werden, selbst wenn man zwei niederenergetische Gravitonen zusammenbringt. Ist in der Quantenelektrodynamik mit Photonen auch so – aber da lassen sich diese Unendlichkeiten wegzaubern, bei der Gravitation geht das nicht.

  100. #100 Panthauer
    19. April 2019

    Darf es deshalb keine „unendliche Energie“ haben, weil dann die „Rechnung/Gleichung“ nicht aufgeht oder weil es verheerende reale Auswirkungen haben würden?
    Wenn ich die Grafik richtig deute, dann vergeht doch „für den ruhenden Beobachter“ keine Zeit, während das Graviton das alles macht. Für das Graviton hingegen, scheint die Zeit unendlich zu sein. Es „bewegt“ sich da nur auf der Orts-Achse, nicht aber auf der Zeit-Achse. Also was „kümmert“ da ein „virtuelles (in der Tat) schwarzes Loch“, wenn es quasi „keine Zeit“ hatte „real“ zu werden?

    Naja, ich beschäftige mich erst seit ein paar Tagen mit den Quanten – und es sind nicht mal meine eigenen…..

  101. #101 MartinB
    19. April 2019

    @Panthauer
    “Darf es deshalb keine „unendliche Energie“ haben, weil dann die „Rechnung/Gleichung“ nicht aufgeht oder weil es verheerende reale Auswirkungen haben würden?”
    Die Gleichungen sagen uns, dass jeder Austausch von Gravitonen sich unendlich stark aufschaukeln würde, das ist das Problem. Das ist in der Realität nicht so, also passen Theorie und Beobachtung nicht zusammen.

    “Wenn ich die Grafik richtig deute, dann vergeht doch „für den ruhenden Beobachter“ keine Zeit, während das Graviton das alles macht. Für das Graviton hingegen, scheint die Zeit unendlich zu sein. ”
    Ich weiß nicht, welche Grafik du da interpretierst, aber du interpretierst falsch. Aussagen wie “für ein Graviton vegeht keine Zeit” sind physikalisch sinnlos – Gravitonen sind keine physikalischen Beobachterinnen.

  102. #102 Panthauer
    19. April 2019

    @MartinB
    “Ich weiß nicht, welche Grafik du da interpretierst, aber du interpretierst falsch.”
    https://scienceblogs.de/hier-wohnen-drachen/files/2013/08/Onephoton.png
    https://scienceblogs.de/hier-wohnen-drachen/files/2013/08/OnephotonPol.png
    etc.
    Es ist trotzdem verwirrend: wenn sich ein virtuelles Teilchen in der Ortsachse nicht bewegt, dann hieße das ja, dass es nie “den Ort seiner Geburt” verlässt, während eine gewisse Zeit vergeht……?

    Zu dem „Fazit“:
    Wenn ich jetzt mal versuche weiter zu denken, dann fällt mir spontan die „Hawking-Strahlung“ ein:
    https://urknall-weltall-leben.de/images/Hawkingstgrahlung.png
    könnte diese nicht ausreichen, um das super kleine „Schwarze Loch“ sofort zu vernichten und eben als „neues Teilchen“ weiter zu existieren? Ich meine: das Teilchen, das vom SL verschluckt wird wäre weg (ebenso das SL) und das andere Teilchen würde weiter „fliegen“…..vielleicht würde das “verschluckte Teilchen” den “Entstehungsprozess” des SL ja umkehren, bis es weg ist – keine Ahnung wie, aber halt mal laut gedacht.

  103. #103 MartinB
    19. April 2019

    Wie kommst du drauf, dass sich ein virtuelles teilchen nicht bewegt? Und beim Interpretieren immer dran denken, dass die Diagramme schematisch zu verstehen sind, die echte Physik gibt es erst, wenn du alle Mögloichkeiten aufaddierst, einzelne Diagramme haben praktisch keine physikalische Bedeutung.

    ” keine Ahnung wie, aber halt mal laut gedacht.”
    Ja, aber weitergedacht funktioniert das halt nicht…

  104. #104 Panthauer
    19. April 2019

    @MartinB
    “Wie kommst du drauf, dass sich ein virtuelles Teilchen nicht bewegt?”
    Ich habe z. B. das hier
    https://scienceblogs.de/hier-wohnen-drachen/wp-content/blogs.dir/69/files/2012/06/i-e6740803b60f6ed561dd3d802ef7be29-feynman0.jpg
    so gelesen, dass sich das Photon (wenn man die Wellenlinie mittelt), auf einer geraden Linie zur Ortsachse “bewegt” bzw. an einem “Punkt” auf der Ortsachse verharrt, also “still steht”. Es vergeht nur Zeit. Ich habe das wohl falsch interpretiert?

    Allerdings – und das verwirrt mich am meisten – sieht es in manchen Feynman Diagrammen so aus, als ob sich ein Teilchen sogar “in der Zeit rückwärts bewegt” und somit theoretisch mit einem anderen Teilchen kollidieren kann „ehe es entstanden ist“. Auch in diesem Fall hätte es sich nicht mal zu bewegen brauchen……….
    Davon abgesehen, würde aus der Sicht eines Teilchens, dass sich mit Lichtgeschwindigkeit bewegt (Photon), eh keine Zeit vergehen und alles gleichzeitig geschehen. Letzteres ist das einzige, was mich nicht irritiert und mich vermuten läßt, dass aus genau diesem Grund das “Graviton” so “problematisch” ist:
    Ein Teilchen das sich mit Lichtgeschwindigkeit bewegt aber gleichzeitig eine Masse hat?

  105. #105 MartinB
    20. April 2019

    Wie gesagt, du nimmst die Diagramme zu wörtlich, es gibt hier nen Haufen Artikel dazu, ich bin aber gerade unterwegs…

  106. #106 Josef Münz
    63846 Laufach
    27. April 2020

    Wenn das virtuelle Graviton mit sich selbst reagiert und auch sein eigenes Anti-Teilchen ist, löscht es sich auch paarweise wieder aus.
    Das sollte zum Kollaps der Überlagerungen führen, zu ihrer Umwandlung in Entropie und so zu einem ungeraden, asymmetrischen, kausalen Zeitpfeil, so daß nur ein virtuelles Graviton übrig bleibt, das dann aber auch wieder verschwindet.
    Das wäre z.B. ein Grenzübergang von der Quantentheorie zur ART, ohne Schwarzes Loch.

  107. #107 MartinB
    27. April 2020

    @Josef Münz
    Nein, wenn man das durchrechnet, führt die Selbstwechselwirkung nicht zu den Phänomenen, die du aus irgendeinem Grund postulierst. Ich sehe auch nicht, wie das mathematisch oder physikalisch gehen sollte.

  108. #108 Josef Münz
    28. April 2020

    Hallo, Guten Morgen.
    Schade – wenn das so mathematisch und physikalisch nicht geht, zugleich aber auch nicht renormierbar ist, was ist es dann? Vielleicht der Urknall? Oder es gibt einfach keine Gravito-nen? Auch in dieser Selbstwechselwirkung muss es irgendwo einen Ereignishorizont geben.

  109. #109 MartinB
    28. April 2020

    @Josef Münz
    Wie soll Quantengravitation der Urknall sein?
    Und was soll ein Ereignishorizont damit zu tun haben?
    Es hat nicht wirklich viel Sinn, einfach Schlagworte in die Gegend zu werfen, auch wenn man zugegebenermaßen beim Lesen populärwissenschaftlicher Bücher manchmal diesen Eindruck bekommt.
    Was man tun muss, ist begründete Ideen/Hypothesen aufzustellen und die dann (durch mathematische Berechnung und Vergleich mit dem Experiment) zu prüfen. Siehe auch hier:
    https://scienceblogs.de/hier-wohnen-drachen/2015/07/28/physik-anders-als-sie-aussieht/?all=1

  110. #110 Christian
    Wien
    13. Februar 2023

    Aus dem obigen Blog: “Und jedes weitere Graviton in dem Diagramm kann ja selbst auch ne sehr hohe Energie haben, wird also mit hoher Wahrscheinlichkeit noch weitere Gravitonen aussenden – am Ende bekommen wir einen unendlichen Wirrwarr von unendlich vielen Gravitonen, die sich vermutlich blitzartig in ein schwarzes Loch verwandeln würden.”

    Das klingt nicht nur nach einem sich selbst verstärkenden Prozess, sondern nach einer wunderbaren Energievermehrung, also einem Perpetuum Mobile erster Art! Martin, verstehe ich es richtig, dass du von einem einzelnen Graviton mit endlichem Impuls und endlicher Energie ausgehst, und dann gäbe es ein exponentielles Wachstum bei beiden Parametern, das erst durch die Bildung eines Schwarzen Lochs beendet wird?

    Was ist der entscheidende Faktor, der so einen Vorgang bei Photonen nicht aufkommen lässt? Die Bipolarität beim Elektromagnetismus oder sonst noch etwas?

  111. #111 MartinB
    13. Februar 2023

    @Christian, ja diese Explosion ist das Problem. Bei Photonen passiert das nicht, weil sie selbst ungeladen sind.

  112. #112 Christian
    Wien
    13. Februar 2023

    Aus deinem Blog: “Gravitonen sind selbst ja auch Teilchen, und sie tragen auch eine Energie und einen Impuls. Das bedeutet, dass wir auch die Kopplung von Gravitonen als sich selbst berücksichtigen müssen – Gravitonen können weitere Gravitonen erzeugen. (Photonen können das nicht, weil sie ungeladen sind.)”

    Mit “Photonen können das nicht, weil sie ungeladen sind” meinst du also elektrisch ungeladen, während Gravitonen, so wie Photonen auch, mit kinetischer Energie, also einem Ruhemassenäquivalent (und sonst auch nichts) “geladen” sind. Mit Masse sind alle anderen Austauschteilchen zwar auch “geladen”, aber keines dieser anderen Austauschteilchen trägt jene Ladung in sich, deren Kraft es vermittelt. Es ist daher zwar gut, dass Gravitonen keine Ruhemasse haben, gleichzeitig wäre es aber auch notwendig, dass die lichtschnellen Gravitonen keine kinetische Energie und somit ein Ruhemassenäquivalent aufweisen. Stimmt das so?

    Falls das also so stimmt, dann fehlt im physikalischen Gedankengebäude entweder eine Möglichkeit, Teilchen vorzusehen, die weder eine Ruhemasse noch eine kinetische Energie haben (obwohl sie sich bewegen), oder das Konzept der Gravitation ist falsch.

    Ich weiß, dass du das Gummituchmodell nicht magst, aber ich meine, dass es das Wesen eines Modells ist, dass es manche Aspekte wirklichkeitsgetreu wiedergibt und andere nicht. (Beispielsweise stellen plane Landkarten auch ein Modell der Wirklichkeit dar, und um eine korrekte Vorstellung von der Wirklichkeit zu bekommen, benötige ich drei verschiedene Typen von Landkarten, nämlich eine winkeltreue, eine längentreue und eine flächentreue. Oder ich schaue mir einen Globus an, aber angenommen ich wäre ein zweidimensionales Wesen, dann wäre ich ausschließlich auf die drei Landkartentypen angewiesen.) Ich stelle mir also das Gummituch in der Internationalen Raumstation ISS vor. Wenn ich dort das Gummituch und Stahlkugeln in Berührung bringe, kommt es klarerweise zu keiner Eindellung des Gummituchs. Eine Kritik am Gummituchmodell ist ja, dass es Gravitation mit Gravitation erklärt, was nicht stimmt, denn ich könnte ja in der ISS eine magnetische Platte mit etwas Abstand unter/über dem Gummituch positionieren, und schon würden die Stahlkugeln das Gummituch eindellen. Hier wird also die Gravitationskraft über eine andere Kraft, nämlich eine magnetische erklärt. (Ich weiß schon, dass die Potentialtopfform hier nicht stimmt, aber es lässt sich sicher eine hypothetische Kraft kreieren, die korrekt geformte Potentialtöpfe verursacht.)

    Was ich damit sagen will: Scheint es dir nach 100 Jahren erfolgloser Bemühungen um die Vereinheitlichung von relativistischer Physik und Quantenphysik nicht auch möglich, dass die Gravitation nur eine Scheinkraft ist, die von einer uns unbekannten Kraft verursacht wird – analog zum obigen Gummituchmodell? Diese uns unbekannte Kraft bräuchte nicht einmal unipolar zu sein – wieder analog zum obigen Gummituchmodell. (Wenn wir in einem Fahrzeug in eine Kurve einfahren, dann spüren wir auch eine zusätzliche Gravitation, aber von dieser Gravitation wissen wir sogar, dass es sich um eine Scheinkraft handelt.) Mit insgesamt drei Raumrichtungen ließe sich dann wohl nicht mehr das Auslangen finden, aber das Vorhandensein von bloß drei Raumrichtungen halte ich, wie schon öfter in “Hier wohnen Drachen” kommentiert, ohnehin für unwahrscheinlich.

    Genauso wie von der Relativitätstheorie beschrieben, bräuchte es daher keiner Gravitonen, ja es gibt sie angesichts der oben erwähnten hundertjährigen Schwierigkeiten in der Theoretischen Physik wahrscheinlich auch gar nicht, sondern es könnte stattdessen wohl eher Austauschteilchen einer noch unbekannten Wechselwirkung geben.

  113. #113 MartinB
    13. Februar 2023

    “Es ist daher zwar gut, dass Gravitonen keine Ruhemasse haben, gleichzeitig wäre es aber auch notwendig, dass die lichtschnellen Gravitonen keine kinetische Energie und somit ein Ruhemassenäquivalent aufweisen. Stimmt das so?”
    Nein. Gravitonen müssen eine Energie tragen, das fordern die Grundregeln der Quantenmechanik. E=h f muss für alle masselosen Quantenteilchen gelten.

    Deswegen fehlt da nichts.

    Das Problem ist, dass die Quantengravitation schlicht diese Unendlichekeiten auswirft, man braucht also einen Weg, die zu umgehen. Der könnte z.B. darin bestehen, dass man den Raum in gewisser Weise quantisiert (wie z.B. bei der Loop-Quantengravitation), dann lassen sich diese Unendlichkeiten möglicherweise in den Griff bekommen.

    Natürlich ist es möglich, dass die Gravitation durch etwas anders zu stande kommt, da gibt es ja z.B. solche Entropie-Gravitations-Ideen. Aber mit bloßen Austauschteilchen einer anderen Art wird das nicht gehen, dazu sind die Randbedingungen, die eine Quantenfeldtheorie stellt, zu strikt. (Jedes Austauschteilchen, das Gravitation beschreiben kann, muss Spin 2 sein und ist damit schon das Graviton).

    Nochmal was Generelles: PhysikerInnen sind nicht doof. Alle Ideen, die man als Laie so haben kann, von der Art “Könnte es nicht sein” hatte auch garantiert schon mal ein Profi und hat sie dann verworfen, weil sie nicht funktioniert.

  114. #114 Christian
    Wien
    13. Februar 2023

    Du missverstehst: “Gravitonen müssen eine Energie tragen, das fordern die Grundregeln der Quantenmechanik.”

    Ich lag also richtig damit, dass die lichtschnellen Gravitonen keine kinetische Energie und somit ein Ruhemassenäquivalent aufweisen dürften, wenn sie als brauchbare Austauschteilchen ohne diese exponentielle Vermehrungsrate herhalten sollen. Da sie, wie du schreibst “Energie tragen” und mir diese Annahme auch bekannt ist, habe ich in meinem vorigen Komnentar #112 eben die Möglichkeitsform, nämlich “wäre” gewählt. Denn wir wissen ja alle, dass diese, wie du schreibst, “Explosion” von Gravitonen in der Natur nicht verwirklicht ist. Also ist irgendetwas am Konzept der Gravitation falsch.

    Martin: “Jedes Austauschteilchen, das Gravitation beschreiben kann, muss Spin 2 sein und ist damit schon das Graviton.”

    Nein. Wenn die Gravitation eine Scheinkraft ist, als die sie in der Relativitätstheorie auch gedeutet wird, dann nicht. Da weist du offenbar eine gewisse quantenphysikalisch getriggerte Betriebsblindheit auf.

    Martin: “Alle Ideen, die man als Laie so haben kann, von der Art “Könnte es nicht sein” hatte auch garantiert schon mal ein Profi und hat sie dann verworfen, weil sie nicht funktioniert.”

    Wenn mir jetzt gelegen wäre, mit dir zu streiten, dann würde ich sogar in der jüngeren Physikgeschichte wohl mindestens ein Gegenbeispiel finden, dass deine verallgemeinernde Aussage (“Alle Ideen …”) widerlegt.

    Martin: “Natürlich ist es möglich, dass die Gravitation durch etwas anders zu stande kommt, da gibt es ja z.B. solche Entropie-Gravitations-Ideen.”

    Eben, bleibe also sachlich und konstruktiv und komm’ da nicht als beleidigte Leberwurst daher: “PhysikerInnen sind nicht doof.” Physiker sind weder doof, noch fehlerfrei – wie sich anhand der hundertjährigen vergeblichen Suche nach der Quantengravitation erkennen lässt.

  115. #115 MartinB
    13. Februar 2023

    “Ich lag also richtig damit, dass die lichtschnellen Gravitonen keine kinetische Energie und somit ein Ruhemassenäquivalent aufweisen dürften, wenn sie als brauchbare Austauschteilchen ohne diese exponentielle Vermehrungsrate herhalten sollen. ”
    Nein. Austauschteilchen ohne, wie du es nennst, Ruhemassenäquivalent kann es nicht geben.

    “Wenn die Gravitation eine Scheinkraft ist, als die sie in der Relativitätstheorie auch gedeutet wird, dann nicht.”
    Dann erklär mal, wie das gehen soll, z.B. mit Gravitationswellen 8die ja den Spin-2-Charakter implizieren).

    ” dann würde ich sogar in der jüngeren Physikgeschichte wohl mindestens ein Gegenbeispiel finden, dass deine verallgemeinernde Aussage (“Alle Ideen …”) widerlegt.”
    Da bin ich gespannt. Kenne die Geschichte ganz gut, mir fällt da wenig ein. (Am ehesten noch der Mpemba-Effekt, aber das ist nichts wirklich fundamentales.)

  116. #116 Christian
    Wien
    13. Februar 2023

    Welche Eigenschaften hätte das Austauschteilchen einer hypothetischen Wechselwirkung, die mindestens bipolar ist, bei der sich aber gleichnamige Ladungen (“Ladungen” meine ich hier allgemein, nicht speziell elektrisch) anziehen und ungleichnamige abstoßen. Hätte das Austauschteilchen unabhängig von der Polanzahl der Wechselwirkung immer Spin 2?

  117. #117 MartinB
    13. Februar 2023

    Eine Wechslewirkung, bei der sich gleichnamige Ladungen anziehen und es trotzdem ungleichnamige gibt, die sich abstoßen, kann es soweit ich sehe in der QFT nicht geben. Ein masseloses Spin-2-Teilchen muss an die Energiedichte koppeln – o.k. vielleicht kann man irgendeine Theorie mit negativen Massen/Energiedichten hinbasteln, aber das ist ziemlich schwierig:
    https://en.wikipedia.org/wiki/Negative_mass

    Generel lgilt: Austauschteilchen mit ungeradzahligem Spin (1,3,5) gehören zu einer Wechselwirkung, wo sich entgegengesetzte Ladungen anziehen, Austauschteilchen mit geradzahligem Spin (2,4,6) zu einer, wo sich gleiche Ladungen anziehen. (Das erkläre ich in meiner QFT-Serie, irgendwo zwischen Teil 10 und 15 oder so…) Hinzu kommt das Weinberg-Theorem, das sagt, dass zu jedem Spin-1-Teilchen eine Erhaltungsgröße als Ladung gehört, und dass ein Spin-2-teilchen zwingend an die Energiedichte koppeln und sich wie ein Graviton verhalten muss.

    Und ein Austauschteilchen, dass die Gravitation beschreibt, muss eh Spin 2 haben, damit im klassischen Grenzfall die richtige Symmetrie von Gravitationswellen rauskommt und weil nur damit eine Theorie entsteht, bei der die ART als Grenzfall rauskommt.

    Das ist ja genau das Problem: Man weiß sehr genau, wie eine QG aussehen müsste – die Theorie ist nur leider nicht renormierbar und damit wenig hilfreich.

  118. #118 Christian
    Wien
    13. Februar 2023

    “Austauschteilchen mit geradzahligem Spin (2,4,6) zu einer, wo sich gleiche Ladungen anziehen.”

    Ich habe es befürchtet. Siehst du dann irgendeine Möglichkeit, die Gravitation als Direktkraft (also nicht als Scheinkraft) festzumachen? So viele Physik-Experten und Physik-Laien 😉 haben es versucht und sind gescheitert. “Natürlich ist es möglich, dass die Gravitation durch etwas anders zu stande kommt.” Ich halte die Gravitation als Scheinkraft nach wie vor für die wahrscheinlichste Variante.

    “Dann erklär mal, wie das gehen soll, z.B. mit Gravitationswellen, die ja den Spin-2-Charakter implizieren.”

    Dazu wäre es wohl notwendig, passende Eigenschaften für eine Wechselwirkung zu finden (auch was die Anzahl der Raumrichtungen betrifft, innerhalb der sie auftritt), die die Gravitation als Scheinkraft überhaupt hervorufen kann. Das geht nicht auf die schnelle, da fängt man wieder bei Null an.

    Danke für deine insgesamt interessanten Blogs und deine Rückmeldungen hier, jedenfalls die fachlich relevanten! Es war eine interessante Diskussion heute mit dir!

  119. #119 MartinB
    14. Februar 2023

    @Christian
    “Das geht nicht auf die schnelle, da fängt man wieder bei Null an.”
    Sag ich doch.

    Wäre übrigens wirklich an Beispielen interessiert, wo Laien in den letzten 50 oder 100 Jahren einen signifikanten Beitrag zur (theoretischen) Physik geleistet haben.

  120. #120 Christian
    Wien
    14. Februar 2023

    “Wäre übrigens wirklich an Beispielen interessiert, wo Laien in den letzten 50 oder 100 Jahren einen signifikanten Beitrag zur (theoretischen) Physik geleistet haben.”

    Manfred von Ardenne (ohne akademischen Grad) fällt mir da auf die schnelle ein. Der war sehr produktiv, auch auf dem Gebiet der Kernphysik. Dass er beim Physik-Nobelpreis 1986 für die Entwicklung der Elektronenmikroskopie neben Ernst Ruska, Gerd Binnig und Heinrich Rohrer nicht berücksichtigt wurde, hatte Anlass zur Kritik gegeben.

    Ansonsten müsste man zu dem Thema recherchieren, weil ein Buch mit einem Titel wie etwa “Was Laien für die Naturwissenschaften geleistet haben” habe ich jetzt nicht ergoogeln können.

    Mehr Zeit beim Googeln werde ich mir für dein obiges Stichwort “Entropie-Gravitations-Ideen” geben.

  121. #121 MartinB
    14. Februar 2023

    @Christian
    Dass das in der Angewandten Physik eher möglich ist, glaube ich sofort (ist aber auch schon 100 Jahre her).
    Es gibt viele Disziplinen, wo jemand ohne akademischen Abschluss wertvolle Beiträge leisten kann, z.B. ist in der Paläontologie Greg Paul eine feste Größe, der hat meines Wissens nie studiert, sich aber einen Namen gemacht.

    Ich habe aber Zweifel, dass das in der theoretischen Physik möglich ist – es sei denn, jemand macht ein komplettes Studium quasi als Autodidakt, aber auch das dürfte sehr hart sein.

  122. #122 Christian
    Wien
    17. Februar 2023

    Während Mordehai Milgroms Modifizierte Newton-Dynamik eine empirische Lösung für die beobachteten Winkelgeschwindigkeiten in den Randbereichen von Galaxien ist, sind die f( R)- bzw. Chamäleon-Gravitation und die Entropische Gravitation um diese zusätzliche Beobachtung herum entwickelte Theorien, wobei letztere angesichts der im Blog geschilderten Probleme vernünftigerweise sogar ohne Gravitonen auskommt, dies aber nicht ad hoc, sondern mit dem 2. Hauptsatz der Thermodynamik begründet. Beide Theorien kommen auch ohne Dunkler Energie und Dunkler Materie aus, was ebenso sinnvoll ist, weil die Dunkle Materie schon im Jahr 1933 von Fritz Zwicky ins Spiel gebracht wurde, aber bis heute genauso nicht verifiziert werden konnte, wie die oben angesprochenen Ideen für eine Quantengravitation.

    Noch etwas zur Allgemeinen Relativitätstheorie: John Archibald Wheeler wird der folgende Satz zugeschrieben: “Der Raum sagt der Materie, wie sie sich bewegen soll, und die Materie sagt dem Raum, wie er sich krümmen soll.”
    Ist das ein Zirkelschluss? Zumindest bleibt die ART in dem Punkt deskriptiv. Es wird zwar festgestellt, dass Materie die Raumzeit krümmt, aber nicht warum. Es könnte wie gesagt eine unbekannte Wechselwirkung sein, die Materie dazu veranlasst, die Raumzeit zu krümmen. Eine solche Wechselwirkung hätte dann Austauschteilchen. Austauschteilchen lassen sich durch Energiezufuhr in reelle Zustände übertragen. Hat sich in Teilchenbeschleunigern schon ein passender Kandidat für ein solches Austauschteilchen finden lassen? Naja, immerhin finden sich Seiten im Internet mit Überschriften wie “CERN: Hinweise auf fünfte physikalische Kraft”. Es besteht also Hoffnung.

  123. #123 MartinB
    17. Februar 2023

    @Christian
    Zum zweiten Absatz: Nein, das ist kein Zirkelschluss, das ist in der Physik ganz normal. Nimm die Maxwellgleichungen: Ladungen erzeugen Felder, Felder bewegen Ladungen, das ist genau dasselbe. In der guten alten Newtonschen Gravitation (als Feldtheorie aufgefasst) ist das auch so.

    Es gibt wirklich keinen Grund, zu versuchen, die ART irgendwie geheimnisvoller zu machen als sie ist…

  124. #124 Christian
    Wien
    22. Februar 2023

    Martin: “Austauschteilchen ohne, wie du es nennst, Ruhemassenäquivalent kann es nicht geben.”

    Abgesehen davon, dass ich aufgrund der Sachlage eben davon ausgehe, dass die Gravitation eine Scheinkraft ist und es daher auch kein Austauschteilchen der Gravitation gibt, bitte ich dich, zu erläutern, wie sich das Ruhemassenäquivalent eines Gravitons errechnet. Für Photonen braucht es dafür zwei Konstanten und die dem Photon zugehörige Wellenlänge: m = h / λc
    (Im übrigen würden Photonen wegen der obigen Beziehung wohl auch virtuelle Gravitonen austauschen, womit es auch keine elektromagnetischen Wellen gäbe.)

    Anhand der Animationen von Markus Pössel zeigst du, wie Gravitationswellen die Raumzeit verzerren. Welche Eigenschaften müsste die Raumzeit haben, um Gravitationswellen (und gegebenfalls auch elektromagnetische Wellen) als Schallwellen der Raumzeit zu deuten?

  125. #125 MartinB
    22. Februar 2023

    @Christian
    Auch für Gravitonen gilt E=h f.
    Und ja, Photonen tauschen in der QG virtuelle Gravitonen aus. Das Photonen auf Schwerefelder reagieren, wissen wir ja.

    Man kann Gravitationswellen nicht als Schallwellen deuten, insofern ergibt die Frage keinen Sinn. Ich kann mir jedenfalls kein Modell der Gravitation vorstellen, das mit einer Schallanalogie funktioniert.

  126. #126 Christian
    Wien
    23. Februar 2023

    Dass sich elektromagnetische und gravitative Wellen in allen Richtungen und unabhängig von der Bewegungsgeschwindigkeit der Wellenquelle oder des Wellenempfängers mit Lichtgeschwindigkeit ausbreiten, liegt bloß daran, weil sich Quelle und Empfänger gegenseitig längenkontrahiert und zeitdilitiert wahrnehmen. Wie sieht das ganze aber aus einer Superposition aus, also wenn beispielsweise ich als dreidimensionaler Beobachter aus der Senkrechten eine zweidimensionale Welt betrachte? Ich gehe davon aus, dass ich die Szenerie aus dieser Position klassisch physikalisch in der Art einer Schallausbreitung sehe. Warum tue ich das deiner Meinung nach nicht?

  127. #127 MartinB
    24. Februar 2023

    @Christian
    Ich verstehe deine Frage nicht. Was genau willst du wie aus 3 Dimensionen in einer 2D-Welt betrachten?
    Mal davon abgesehen, dass die Formulierung “liegt bloß daran, dass” zwar nicht wirklich falsch ist, aber mMn am Kern der RT massiv vorbeigeht.

  128. #128 Christian
    Wien
    24. Februar 2023

    Martin: “Man kann Gravitationswellen nicht als Schallwellen deuten …”

    Eine entsprechende Google-Suche zeigt, dass die Analogie von Gravitations- und Schallwellen auch anderen aufgefallen ist:

    “Da sie aus Vibrationen in der Struktur der Raumzeit bestehen, lassen sich Gravitationswellen mit Schallwellen vergleichen.” https://www.spektrum.de/news/6-fragen-die-uns-gravitationswellen-beantworten-koennten/1398802

    Hier gibt es eine Erläuterung zu Gemeinsamkeiten und Unterschieden von Gravitations- und Schallwellen: Analogieversuche zur
    Gravitationswellendetektion, http://www.physik.kit.edu/Studium/Lehramt/LIGO_WS_20_21.pdf

    Warum die Geschwindigkeit der Gravitationswellen gegenüber Schallgeschwindigkeiten in Medien sehr viel größer ist, leuchtet anhand der immensen Steifigkeit der Raumzeit auch ein. Etwa hier: https://www.philoclopedia.de/einzeldisziplinen/astronomie/gravitationswellen/

  129. #129 MartinB
    24. Februar 2023

    Natürlich lassen sich Wellen vergleichen, das heißt aber nicht, dass man das eine als das andere interpretieren kann.

    Und das mit dem Elastizitätsmodul der Raumzeit liest man zwar oft, aber es ist mMn wirklich absolut falsch:
    https://scienceblogs.de/hier-wohnen-drachen/2018/08/25/ist-die-raumzeit-elastisch/?all=1